LIVE Test – 2 21st march 2020

Q1. Pro-Active Governance and Timely Implementation (PRAGATI), is set up under the a. Department of Personnel and Training, Union Ministry of Personnel, Public Grievances and Pensions b. NITI Aayog c. Cabinet Secretariat d. Prime Minister’s Office Answer: d Explanation: PRAGATI stands for Pro-Active Governance and Timely Implementation is a multi- purpose and interactive ICT platform with the objective of initiating a culture of Pro-Active Governance and Timely Implementation by addressing common man’s grievances, and monitoring important programs of Union and State Governments. It has been designed in- house by the PMO team with the help of National Informatics Center (NIC). It is a three- tier system (involving PMO, Union Government Secretaries, and Chief Secretaries of the States); PM will hold a monthly interaction with the Government of Secretaries and Chief Secretaries through Video-conferencing enabled by data and geo-informatics visuals; Issues to be raised before the PM are picked up from the available database regarding Public Grievances, on-going Programs and pending Projects. It was launched in 2015. Hence, option (d) is the correct answer. Source: https://vajiramias.com/current-affairs/pro-active-governance-and-timely- implementation-pragati/5d4272cc1d5def4eeb96466f/ Q2. Which of the following pair/s is/are correctly matched?

Protocols/ Initiatives Intention

1. Bonn Challenge to conserve migratory species within their migratory ranges. 2. Montreal Protocol to protect the ozone layer.

3. Washington Convention to protect endangered plants and animals.

Select the correct answer using the code given below. a. 2 only b. 1 and 2 only c. 2 and 3 only d. 1, 2 and 3 Answer: c Explanation: Pair (1) is not correctly matched: The Bonn Challenge is a global movement launched on September 2, 2011 at a ministerial conference in Bonn to restore 150 million hectares of degraded and deforested land by 2020 and 350 million hectares by 2030. In January 2019, India became part of the “Bonn Challenge”. Union Environment Minister has recently announced that India has committed to rejuvenate 50 lakh hectares (5 million) of degraded land between 2021 and 2030. The Convention on the Conservation of Migratory Species of Wild Animals, also known as the Convention on Migratory Species (CMS) or the Bonn Convention, is an international agreement that aims to conserve migratory species within their migratory ranges. Pair (2) is correctly matched: Montreal Protocol on substance that deplete the Ozone layer in an international treaty designed to protect the ozone layer by phasing out the production of numerous substances that are responsible for ozone depletion for example chlorofluorocarbons (CFCs) that are present in our air-conditioners and refrigerators. Pair (3) is correctly matched: The Washington Convention on International Trade in Endangered Species of Wild Fauna and Flora, more commonly known as CITES, aims at regulating the international trade of specimens of endangered animals and plants. Source: https://vajiramias.com/current-affairs/land- degradation/5d6625b11d5def0e834a70ff/ Q3. Which of the following nutrients are used by the Government of India to provide Nutrient Based Subsidy on an annual basis? 1. Nitrogen 2. Sulphur 3. Potash 4. Phosphate Select the correct answer using the code given below: a. 1 and 4 only b. 3 and 4 only c. 1, 2 and 3 only d. 1, 2, 3 and 4 Answer: d Explanation: The Cabinet Committee on Economic Affairs (CCEA) has recently approved the proposal of the Department of Fertilizers for fixation of Nutrient Based Subsidy Rates for P& K Fertilizers for the year 2019-20.The Department of fertilizers is implementing Nutrient Based Subsidy Policy for P&K fertilizers since 2010. Under the NBS Policy, a fixed rate of subsidy (in Rs. per Kg basis) is announced on nutrients namely Nitrogen (N), Phosphate (P), Potash (K) and Sulphur (S) by the Government on annual basis.The per Kg subsidy rates on the nutrient N, P, K, S is converted into per Tonne subsidy on the various P& K fertilizers covered under NBS Policy.Any variant of the fertilizers covered under the subsidy scheme with micronutrients namely Boron and Zinc, is eligible for a separate per tonne subsidy to encourage their application alongwith primary nutrients. Hence, option (d) is the correct answer. Source: https://vajiramias.com/current-affairs/nutrient-based-subsidy- nbs/5d4294961d5def4ee9847d1e/ Q4. With reference to the Energy Efficiency Services Ltd (EESL), consider the following statements: 1. It is a joint venture of The Energy and Resources Institute (TERI) and Ministry of New and Renewable Energy (MNRE). 2. It is a Super Energy Service Company (ESCO) and leads the market-related activities of the National Mission for Enhanced Energy Efficiency (NMEEE). Which of the statement/s given above is/are correct? a. 1 only b. 2 only c. Both 1 and 2 d. Neither 1 nor 2 Answer: b Explanation: Statement 1 is not correct: EESL is a joint venture of four national Public-Sector Undertakings: NTPC Limited, Power Finance Corporation Limited, Rural Electrification Corporation Limited and POWERGRID Corporation of India Limited. It is governed under the Union Ministry of Power. Statement 2 is correct: Energy Efficiency Services Ltd (EESL) is a Super Energy Service Company (ESCO) and leads the market-related activities of the National Mission for Enhanced Energy Efficiency (NMEEE), one of the eightnational missions under the Prime Minister’s National Action Plan on Climate Change. Source: https://vajiramias.com/current-affairs/energy-efficiency-services-ltd- eesl/5d43da4f1d5def64de4db120/ Q5. Pashmina wool comes from which of the following breeds of the Cashmere goats? 1. Changthangi 2. Malra 3. Chegu Select the correct answer using the code given below. a. 1 only b. 1 and 2 only c. 2 and 3 only d. 1, 2 and 3 Answer: d Explanation: Pashmina is a fine type of Kashmiri wool which comes from 4 distinct breeds of the Cashmere goat namely: 1. Changthangi (or Kashmir Pashmina goat) from the Changthang Plateau in Kashmir region, 2. Malra from Kargil area in Kashmir region, 3. Chegu from Himachal Pradesh 4. Chyangara (or Nepalese Pashmina goat) from Nepal. Pashmina (also known as ‘cashmere’ in the West) literally translates to ‘Soft Gold’ in Kashmiri. It is used to make shawl, scarf and wrap/stole.Kashmir Pashmina has been assigned Geographical indication (GI) tag under the Geographical Indications of Goods (Registration and Protection) Act, 1999. Hence, option (d) is the correct answer. Source: https://vajiramias.com/current-affairs/pashmina-testing- centre/5d43e2ee1d5def64e44568b7/ Q6. Which of the following countries are the members of Mekong Ganga Cooperation (MGC)? 1. Cambodia 2. Vietnam 3. Laos 4. Bangladesh Select the correct answer using the code given below. a. 1 and 4 only b. 1 and 2 only c. 2, 3 and 4 only d. 1, 2 and 3 only Answer: d Explanation: The Mekong Ganga Cooperation (MGC), is a sub-regional cooperation organisation comprising India and five ASEAN countries, namely, Cambodia, Laos, Myanmar, Thailand and Vietnam. MGC was launched in 2000 at Laos’ capital Vientiane. MGC takes its name from the Ganga and the Mekong, the two civilisational rivers in Southeast Asia. The MGC aims at facilitating closer contacts among the people inhabiting these two major river basins and enhancing cooperation in tourism, culture, education, transport and communications. During the recent meeting in Bangkok, the new MGC Plan of Action 2019-2022 was adopted that envisages project-based cooperation in various areas including tourism and culture, education, public health and traditional medicine, agriculture and allied sectors, transport and communication, MSMEs and water resources management. Hence, option (d) is the correct answer. Source: https://vajiramias.com/current-affairs/mekong-ganga-cooperation- mgc/5d454a191d5def64dbd9d7da/ Q7. With reference to the kala-azar disease, consider the following statements: 1. In India, Leishmania Donovani is the only parasite that causes this disease. 2. It is endemic only to , Uttar Pradesh, Kerala and Gujarat in India. Which of the statement/s given above is/are correct? a. 1 only b. 2 only c. Both 1 and 2 d. Neither 1 nor 2 Answer: a Explanation: Visceral Leishmaniasis (VL) also known as kala-azar, black fever, and Dumdum fever is a disease caused by the parasites of the Leishmania type. It is spread by the bite of certain types of sandflies and is classified as a neglected tropical disease. The parasite is spread to humans by bites from infected female sand flies. This parasitic disease is confined to humans. In this, the parasite migrates to the internal organs such as the liver, bone marrow and spleen (hence ‘visceral’). It attacks the immune system, and is almost always fatal if not treated. This disease is the second-largest parasitic killer in the world (after malaria). It has a high fatality rate. If untreated, it can kill within two years of the onset of the ailment. In India, Leishmania Donovani is the only parasite that causes this disease. Under National Vector Borne Disease Control Programme (NVBDCP), Government of India is coordinating with endemic States to eliminate the disease. Hence statement 1 is correct. Kala azar is endemic to the Indian subcontinent in four countries (Bangladesh, Bhutan, India and Nepal). Within India it is endemic to four contagious states of West Bengal, Bihar, Jharkhand and eastern Uttar Pradesh. Hence statement 2 is not correct. Source: https://vajiramias.com/current-affairs/kala-azar-visceral- leishmaniasis/5d4685b41d5def64dfd7644c/ Q8. What is GJ 357 d, recently seen in the news? a. Super-Earth Exoplanet b. Newly discovered Satellite of Saturn c. Halley-type comet near Venus d. Ancient ‘Mega Site’ in Jerusalem Answer: a Explanation: The super-Earth planet -- named GJ 357 d -- was discovered in early 2019 owing to NASA’s Transiting Exoplanet Survey Satellite (TESS), a mission designed to comb the heavens for exoplanets. Astronomers from Spain, announced the discovery of the GJ 357 system in the journal Astronomy & Astrophysics. They showed that the distant solar system — with a diminutive M-type dwarf sun, about one-third the size of our own sun — harbours three planets, with one of those in that system’s habitable zone: GJ 357 d. With a thick atmosphere, the planet GJ 357 d is 22% larger than the Earth and could maintain liquid water on its surface like Earth. Hence, option (a) is the correct answer. Additional Information: Super Earth is an extrasolar planet with a mass higher than Earth’s, but substantially below the masses of the Solar System’s ice giants, Uranus and Neptune. This term refers only to the mass of the planet, and does not imply anything about the habitability or surface conditions. Source: https://vajiramias.com/current-affairs/gj-357-d/5d46898e1d5def64d9b410b9/ Q9. In the context of Pan African e-network (PAeN) project, consider the following statements: 1. It was launched during the Third India-Africa Forum Summit (IAFS-III) at New Delhi. 2. The Department of Space and Telecommunications Consultants India Limited (TCIL) is the implementing agency of the project. Which of the statement/s given above is/are correct? a. 1 only b. 2 only c. Both 1 and 2 d. Neither 1 nor 2 Answer: b Explanation: Pan African e-network (PAeN) project was launched in 2009. It was intended to provide ‘seamless’ and ‘integrated’ satellite, fiber optics and wireless network to connect the African countries. The project has two main components: Tele-Education Services and Tele-Medicine Services. The project is a joint undertaking of the Government of India and the African Union (AU) High Commission.Hence statement 1 is not correct. India’s Ministry of External Affairs is the designated nodal Ministry and The Department of Space and Telecommunications Consultants India Limited (TCIL) is the implementing agency of the project. Hence statement 2 is correct. Source: https://vajiramias.com/current-affairs/india-africa- relations/5d4678d21d5def64da940f20/ Q10. With reference to the Canine Distemper Virus (CDV), consider the following statements: 1. It is caused by a single-stranded RNA virus of the family Paramyxoviridae. 2. Its treatment typically consists of supportive care and efforts to prevent infections. Which of the statement/s given above is/are correct? a. 1 only b. 2 only c. Both 1 and 2 d. Neither 1 nor 2 Answer: c Explanation: Canine distemper is a contagious viral disease that attacks the respiratory, gastrointestinal and nervous systems of a wide variety of animal species, including dogs, coyotes, foxes, pandas and wolves. It is also known as hardpad disease. Common symptoms include high fever, eye inflammation and eye/nose discharge, vomiting and diarrhoea, loss of appetite and lethargy, and hardening of nose and footpads. It is caused by a single-stranded RNA virus of the family Paramyxoviridae(the same family of the viruses that causes measles, mumps, and bronchiolitis in humans). This virus is similar to the measles virus in humans and the rinderpest virus which affects cattle. The disease is highly contagious via inhalation. The virus can also be transmitted by shared food and water bowls and equipment. There is no cure for canine distemper infection. Treatment typically consists of supportive care and efforts to prevent infections. Hence both statements are correct. Source: https://vajiramias.com/current-affairs/canine- distemper/5d4680531d5def64dbda03b8/ Q11. Nar-Par-Tapi linking project, often mentioned in the news, is for providing water for which of the following regions? a. Saurashtra b. Bundelkhand c. Malwa d. Marathwada Answer: d Explanation:

Maharashtra Cabinet has recently approved Nar-Par-Tapiriver linking project to ensure 52 tmcft water for Marathwada and parts of north Maharashtra. The Nar-Par-Tapi linking project, which is a combination of a lift irrigation scheme, tunnels and canals, will ensure 52 thousand million cubic feet (TMCft) water for Marathwada and parts of north Maharashtra. Hence, option (d) is the correct answer. Source: https://vajiramias.com/current-affairs/nar-par-tapi-river-linking- project/5d6a224f1d5def3e370d903c/ Q12. With reference to Saffron, consider the following statements: 1. In Jammu and Kashmir, 90 % of saffron production comes from Ladakh. 2. The National Saffron Mission is implemented under the Rashtriya Krishi Vikas Yojana (RKVY). Which of the statement/s given above is/are correct? a. 1 only b. 2 only c. Both 1 and 2 d. Neither 1 nor 2 Answer: b Explanation: Ladakh is part of saffron farming experiment along with hill States of Himachal Pradesh and Uttarakhand. At present, saffron is farmed in Pulwama and Pampore areas of Jammu and Kashmir. In Kashmir valley, 90 % of saffron production comes from Pampore uplands, locally called ‘wudar’ on the southern outskirts of Srinagar city. Ladakh has similarities of climatic conditions with Iran and Afghanistan where saffron is grown. Hence statement 1 is not correct. Saffron, one of the world’s costliest spice is in great demand for its medicinal, cosmetic and aromatic properties. It grows from a very small plant botanically known as Crocus Sativus. Its purple coloured flower is the only part mostly seen above the ground. The blossoms appear during late autumn (mid-October to early November). The aromatic reddish stigmas of these flowers are harvested which form the most expensive part of the colourful spice called ‘Mogra’. The remaining stalks are also processed to get inferior grade of saffron called ‘Lacha’. Government of India is implementing the National Saffron Mission under the RashtriyaKrishiVikasYojana (RKVY). Hence statement 2 is correct. Source: https://vajiramias.com/current-affairs/saffron/5d46564e1d5def64dfd75e52/ Q13. With reference to the Uchhatar Avishkar Yojana (UAY), consider the following statements: 1. Its objective is to promote innovation of a higher order that directly impacts the needs of the Industry. 2. UAY projects are funded jointly by Ministry of Skill Development and Entrepreneurship and World Bank. Which of the statement/s given above is/are correct? a. 1 only b. 2 only c. Both 1 and 2 d. Neither 1 nor 2 Answer: a Explanation: Uchhatar Avishkar Yojana (UAY) was launched in 2015 with an objective to promote innovation of a higher order that directly impacts the needs of the Industry and thereby improves the competitive edge of Indian manufacturing. A total of 142 projects have been approved under UAY. Hence statement 1 is correct. UAY projects are funded jointly by Ministry of Human Resource Development, participating Ministries and the Industry in the ratio of 50:25:25. Hence statement 2 is incorrect. Source: https://vajiramias.com/current-affairs/imprint-uay/5d477f951d5def64e445fe0d/ Q14. In the context of Quick Reaction Surface-to-Air Missile (QRSAM), consider the following statements: 1. It has a strike range of about 1000 km capable of killing multiple sea based targets. 2. It has been indigenously developed by Defence Research and Development Organisation (DRDO). Which of the statement/s given above is/are correct? a. 1 only b. 2 only c. Both 1 and 2 d. Neither 1 nor 2 Answer: b Explanation: Statement 1 is not correct: The Quick Reaction Surface-to-Air Missile (QRSAM), has a strike range of about 30 km, is capable of killing multiple aerial targets, tanks and bunkers. The systems are equipped with indigenously-developed Phased array radar, Inertial Navigation System, Data Link & RF seeker. The system is being developed for Indian Army with search and track on move capability with very short reaction time. Statement 2 is correct: The missile has been indigenously developed by Defence Research and Development Organisation (DRDO). Recently DRDO successfully flight-tested its state-of-the-art QRSAM against live aerial targets from Integrated Test Range (ITR) at Chandipur, Odisha. Source: https://vajiramias.com/current-affairs/quick-reaction-surface-to-air-missiles- qrsam/5d4781ce1d5def64da9437e8/ Q15. With reference to the Elephant Census, consider the following statements: 1. The census of elephants is undertaken at the national level once every four years. 2. In the IUCN Red List, African elephants are listed as “vulnerable” and Asian elephants as “endangered” species. Which of the statement/s given above is/are correct? a. 1 only b. 2 only c. Both 1 and 2 d. Neither 1 nor 2 Answer: b Explanation: Statement 1 is not correct: The Elephant population in India is estimated at 29,964 as per the census conducted in 2017. The South Region accounted for 14,612 followed by North East with 10,139 elephants. While the census of tigers is undertaken at the national level once every four years, the census of elephants is undertaken at every five years. Statement 2 is correct: African elephants are listed as “vulnerable” and Asian elephants as “endangered” in the IUCN Red List of threatened species. As per the available population estimates, there are about 400,000 African elephants and 40,000 Asian elephants. Source: https://vajiramias.com/current-affairs/elephant- census/5d4786d21d5def64dfd78bf9/ Q16. Which of the following pairs is/are correctly matched?

Region often in the news Countries

1. Chittagong Sri Lanka

2. Goma Myanmar

3. Uluru France

Select the correct answer using the code given below: a. 1 and 2 only b. 2 and 3 only c. 1, 2 and 3 d. None of the above Answer: d Explanation: Pair (1) is not correctly matched: The Chittagong Hill Tracts are an area within the Chattogram Division in southeastern Bangladesh, bordering Myanmar on the southeast, the Indian state of Tripura on the north, Mizoram on the east and Chittagong district on the west. Topographically, the Hill Tracts are the only extensively hilly area in Bangladesh.

Pair (2) is not correctly matched: Goma is the capital of North Kivu province in the eastern Democratic Republic of the Congo.

Pair (3) is not correctly matched:Uluru, or Ayers Rock, is a massive sandstone monolith in the heart of the Northern Territory’s arid "Red Centre" in Australia. Uluru is listed as a UNESCO World Heritage Site.Hence, option (d) is the correct answer.

Source: https://vajiramias.com/current-affairs/chittagong-hill-tract-cht- accord/5d478bfb1d5def64de4e4776/ Q17. With reference to the Genome India Initiative, consider the following statements: 1. It is an initiative taken by the Tata Institute of Fundamental Research. 2. A genome is an organism’s complete set of DNA, including all its genes. Which of the statements given above is/are correct? a. 1 only b. 2 only c. Both 1 and 2 d. Neither 1 nor 2 Answer: b Explanation: Statement 1 is not correct: Under Genome India Initiative, the Department of Biotechnology (DBT), Government of India plans to scan nearly 20,000 Indian genomes over the next 5 years, in a two-phase exercise, and develop diagnostic tests that can be used to test for cancer. Statement 2 is correct: A genome is an organism’s complete set of DNA, including all its genes. It contains all the information needed to build and maintain that organism. By sequencing the genome, researchers can discover the functions of genes and identify which of them are critical for life. Source: https://vajiramias.com/current-affairs/genome-india- initiative/5d479cb61d5def64de4e494a/ Q18.With reference to the Permafrost, consider the following statements: 1. It is any ground that remains completely frozen for at least two decades. 2. A layer of soil on top of permafrost does not stay frozen all year. Which of the statements given above is/are correct? a. 1 only b. 2 only c. Both 1 and 2 d. Neither 1 nor 2 Answer: b Explanation: Statement 1 is not correct: Permafrost is any ground that remains completely frozen—32°F (0°C) or colder—for at least two years straight. These permanently frozen grounds are most common in regions with high mountains and in Earth’s higher latitudes—near the North and South Poles. Permafrost covers large regions of the Earth. Almost a quarter of the land area in the Northern Hemisphere has permafrost underneath. Statement 2 is correct: Permafrost is made of a combination of soil, rocks and sand that are held together by ice. The soil and ice in permafrost stay frozen all year long. Near the surface, permafrost soils also contain large quantities of organic carbon—a material leftover from dead plants that couldn’t decompose, or rot away, due to the cold. Lower permafrost layers contain soils made mostly of minerals. A layer of soil on top of permafrost does not stay frozen all year. This layer, called the active layer, thaws during the warm summer months and freezes again in the fall. Source: https://vajiramias.com/current-affairs/arctic- wildfires/5d4910fa1d5def64d73e8d98/ Q19. Which of the following banks will be a party to amalgamation scheme recently announced by the Government for merger of public sector banks (PSBs) ? 1. Indian Bank 2. Punjab National Bank 3. Andhra Bank 4. Syndicate Bank Select the correct answer using the code given below: a. 2 and 4 only b. 1 and 2 only c. 2 and 3 only d. 1, 2 and 4 only Answer: b Explanation: Government of India has announced the merger of 10 public sector banks (PSBs) into 4 banks. Under the scheme of amalgamation, 1. Indian Bank will be merged with Allahabad Bank (anchor bank - Indian Bank); 2. PNB, OBC and United Bank to be merged (PNB will be the anchor bank); 3. Union Bank of India, Andhra Bank and Corporation Bank to be merged (anchor bank - Union Bank of India); and 4. Canara Bank and Syndicate Bank to be merged (anchor bank - Canara Bank). Hence, option (b) is the correct answer. In place of 27 public sector banks in 2017, now there will be 12 public sector banks after the latest round of consolidation of PSU banks. The merger announcement was followed by an equity infusion move of Rs 55,250 crore in these banks to enable them to grow their loan book. This is a step by government towards building Next-Generation Banks. The consolidation of public sector banks will give them scale. Last year, the government had approved the merger of Vijaya Bank and Dena Bank with Bank of Baroda (BoB) that become effective from April 1, 2019. In 2017, the State Bank of India absorbed five of its associates and the BharatiyaMahila Bank. Source: https://vajiramias.com/current-affairs/bank- consolidation/5d6a286e1d5def3e3647c76d/ Q20. With reference to the United Nations Relief and Works Agency for Palestine Refugees in the Near East (UNRWA), consider the following statements: 1. It is the only UN agency dedicated to helping refugees from a specific region. 2. It is funded almost entirely by voluntary contributions from UN Member States. Which of the statement/s given above is/are correct? a. 1 only b. 2 only c. Both 1 and 2 d. Neither 1 nor 2 Answer: c Explanation: UN Palestine refugee agency was established in 1949, following the 1948 Arab-Israeli conflict. In the absence of a solution to the Palestine refugee problem, the General Assembly has repeatedly renewed UNRWA’s mandate, most recently extending it until 30 June 2020. Its mandate is to support Palestinian refugees, and their patrilineal descendants, who fled or were expelled from their homes during the 1948 Palestine war and 1967 Six Day war. It provides education, health care, and social services to the population it supports. Aid is provided in five areas of operation: Jordan, Lebanon, Syria, the Gaza Strip and the West Bank, including East Jerusalem; Aid for Palestinian refugees outside these five areas is provided by UNHCR. It is funded almost entirely by voluntary contributions from UN Member States. It also receives some funding from the Regular Budget of the United Nations. Hence both statements are correct. Source: https://vajiramias.com/current-affairs/united-nations-relief-and-works-agency- for-palestine-refugees-unrwa/5d4a7dfe1d5def64de4eac75/ Q21. Consider the following statements with reference to the Treaty on the Prohibition of Nuclear Weapons (TPNW),: 1. It is the first legally binding international agreement to comprehensively prohibit nuclear weapons. 2. It was passed immediately after the disintegration of USSR. Which of the statement/s given above is/are correct? a. 1 only b. 2 only c. Both 1 and 2 d. Neither 1 nor 2 Answer: a Explanation: Statement 1 is correct: The Treaty on the Prohibition of Nuclear Weapons (TPNW) is also known as the Nuclear Weapon Ban Treaty. It is the first legally binding international agreement to comprehensively prohibit nuclear weapons, with the goal of leading towards their total elimination. Statement 2 is not correct: It was passed in July 2017. In order to come into effect, signature and ratification by at least 50 countries is required. As of 31 July 2019, 24 nations have ratified the treaty. For those nations that are party to it, the treaty prohibits the development, testing, production, stockpiling, stationing, transfer, use and threat of use of nuclear weapons, as well as assistance and encouragement to the prohibited activities. For nuclear armed states joining the treaty, it provides for a time-bound framework for negotiations leading to the verified and irreversible elimination of its nuclear weapons programme. Source: https://vajiramias.com/current-affairs/the-treaty-on-the-prohibition-of-nuclear- weapons-tpnw/5d4a7a861d5def64dfd7f333/ Q22. The National Handloom Day is observed to commemorate the: a. Swadeshi Movement b. Non-cooperation Movement c. Civil Disobedience Movement d. None of the above Answer: a Explanation: The National Handloom Day is observed annually on 7th August to honour the handloom weavers in the country and also highlight India’s handloom industry. The first National Handloom Day was inaugurated on 7th August 2015 by the Prime Minister, NarendraModi. August 7 was chosen as the National Handloom Day to commemorate the Swadeshi Movement which was launched on this day in 1905 in Calcutta Town Hall to protest against the partition of Bengal by the British Government. The movement had aimed at reviving domestic products and production processes. Hence, option (a) is the correct answer. Source: https://vajiramias.com/current-affairs/5th-national-handloom- day/5d4b87221d5def64e1cb76db/ Q23. Organic Field Effect Transistor (OFET), which was recently seen in the news, aims: a. to take sharp pictures of objects in the sky such as planets, stars and galaxies. b. to instantly detect the charges on the cell walls of bacteria. c. to make high-resolution, microwave-wavelength surveys for study of the cosmic microwave background radiation (CMB). d. to study atmospheric neutrinos and promote research in particle physics. Answer: b Explanation: A team of researchers at the Indian Institute of Technology, Guwahati (IIT-G) has recently developed a portable device to detect bacteria without cell culture or microbiological assays.At present, the detection of bacteria in body fluids is done in laboratories. The new portable device uses an Organic Field Effect Transistor (OFET) to instantly detect the charges on the cell walls of bacteria. The rapid detection kits are like those used for blood sugar monitoring. The OFET is an electronic device that works on the principle that charges in the vicinity of the channels of certain semiconductors can induce a current in them. Thus, the charges on the surface of the bacterium, induces a current in the OFET, which is registered and read. Hence, option (b) is the correct answer. Source: https://vajiramias.com/current-affairs/organic-field-effect-transistor- ofet/5d4b92e41d5def64e4468c07/ Q24. The Dixon Plan of 1950, recently seen in the news, is related to: a. Committee for Human Rights in North Korea. b. Resolution on the Kashmir dispute. c. Peacefully divide and claim colonial land in Africa. d. An American initiative passed in 1948 to aid Western Europe. Answer: b Explanation: The idea of dividing Jammu and Kashmir into two or more parts traces its origin to the Dixon Plan of 1950. Owen Dixon, an Australian jurist chosen by the United Nations to mediate between India and Pakistan on the J& K issue, in his report of September 1950, suggested a package, which did not find acceptance from India. The Plan had assigned Ladakh to India and northern areas and Pakistan-Occupied Kashmir to Pakistan, besides splitting Jammu between the two. It had proposed a plebiscite in the Kashmir valley. Hence, option (b) is the correct answer. Source: https://vajiramias.com/current-affairs/dixon-plan-of- 1950/5d4d16451d5def64d9b4fa83/ Q25.With reference to the Bharat Ratna Awards, consider the following statements: 1. Bharat Ratna recipients rank seventh in the Indian order of precedence. 2. Only India-born citizens are eligible for Bharat Ratna Awards. Which of the statements given above is/are correct? a. 1 only b. 2 only c. Both 1 and 2 d. Neither 1 nor 2 Answer: a Explanation: The President of India, Ram Nath Kovind, recently presented Bharat Ratna Awards to Nanaji Deshmukh (posthumously), Bhupendra Kumar Hazarika (posthumously) and Pranab Mukherjee at Rashtrapati Bhavan. Statement 1 is correct: The Bharat Ratna is the highest civilian award of the Republic of India. Bharat Ratna recipients rank seventh in the Indian order of precedence. It was instituted in 1954. The award is conferred in recognition of exceptional service/performance of the highest order in ‘any field of human endeavour’. The award was originally limited to achievements in the arts, literature, science, and public services, but this was changed in 2011. The award is conferred without distinction of race, occupation, position, or sex. Statement 2 is not correct: Though usually conferred on India-born citizens, the Bharat Ratna has been awarded to one naturalised citizen, Mother Teresa, and to two non-Indians, Pakistan national Khan Abdul Ghaffar Khan and former South African President Nelson Mandela. The original statutes did not provide for posthumous awards but were amended in 1955 to permit them. There is no formal nomination process. The recommendations for the award can only be made by the Prime Minister to the President. A maximum of three nominees can be awarded per year. The first recipients of the Bharat Ratna were C. Rajagopalachari, Sarvepalli Radhakrishnan, and C. V. Raman, who were honoured in 1954. Source: https://vajiramias.com/current-affairs/bharat-ratna/5d4d0c0b1d5def64e31bafdb/ Q26. Which of the following bodies/institutions are headed by the Prime Minister of India? 1. Defence Acquisition Council 2. National Integration Council 3. National Ganga Council 4. National Board for Wildlife Select the correct answer using the code given below. a. 1 and 4 only b. 2 and 3 only c. 2, 3 and 4 only d. 1, 2, 3 and 4 Answer: c Explanation: (1) is not correct: Defence Acquisition Council (DAC) was established in 2001. Chairman of DAC is Defence Minister. Its main objective is overall guidance of the defence procurement planning process and ensuring expeditious procurement of the approved requirements of the Armed Forces. (2) is correct: The National Integration Council (NIC) was constituted in 1961, following a decision taken at a national conference on ‘unity in diversity’, convened by the Central government, at New Delhi. It consisted of the prime minister as chairman, central home minister, chief ministers of states, seven leaders of political parties, the chairman of the UGC, two educationists, the commissioner for SCs and STs and seven other persons nominated by the prime minister. (3) is correct: National Mission for Clean Ganga (NMCG) was registered as a society on 12th August 2011 under the Societies Registration Act 1860.It acted as implementation arm of National Ganga River Basin Authority (NGRBA) which was constituted under the provisions of the Environment (Protection) Act (EPA), 1986. NGRBA has since been dissolved with effect from the 7th October 2016, consequent to constitution of National Council for Rejuvenation, Protection and Management of River Ganga (referred as National Ganga Council). National Ganga Council under chairmanship of Hon’ble Prime Minister of India. The Empowered Task Force (ETF) on river Ganga under chairmanship of Hon’ble Union Minister of Water Resources, River Development and Ganga Rejuvenation. (4) is correct: National Board for Wildlife (NBWL) is a statutory body as it has been constituted under Section 5A the Wildlife Protection Act, 1972. It is the apex body to review all wildlife-related matters and approve projects in and around national parks and sanctuaries i.e. Protected Areas. It is a 47-member board (including the chairman) which usually meets once a year. It is chaired by Prime minister. Source: https://vajiramias.com/current-affairs/defence-acquisition-council- dac/5d4d0d5d1d5def64de4f0ac3/ Q27. With reference to the Women Transforming India (WTI) Awards, consider the following statements: 1. It is being organized by NITI Aayog in collaboration with the United Nations to recognize women entrepreneurs from across India. 2. The Women Entrepreneurship Platform (WEP) is a UN Women initiative to promote and support aspiring women entrepreneurs across the world. Which of the statements given above is/are correct? a. 1 only b. 2 only c. Both 1 and 2 d. Neither 1 nor 2 Answer: a Explanation: The NITI Aayog recently launched the Fourth Edition of Women Transforming India (WTI) Awards. Statement 1 is correct: The Women Transforming India (WTI) Awards is being organized in collaboration with the United Nations to recognize women entrepreneurs from across India. This year’s theme is ‘Women and Entrepreneurship’, in continuation of the theme for WTI Awards 2018. WhatsApp has collaborated with NITI Aayog for WTI Awards 2019 and will be providing support amounting to US$100,000 to the award winners. Statement 2 is not correct: The Women Entrepreneurship Platform (WEP) is a Government of India initiative by NITI Aayog to promote and support aspiring as well as established women entrepreneurs in India, assist and handhold them in their journey from starting to scaling up and expanding their ventures. It has more than 5,000 women entrepreneurs registered on the platform, more than 30 partners and, has committed funding of more than US$10mn for these startups. Source: https://vajiramias.com/current-affairs/women-transforming-india-wti- awards/5d4d16f11d5def64dbdae59b/ Q28. Shillong declaration, recently seen in news, is related to: a. e-Governance b. Intellectual Property Rights c. Violence against women d. Climate Change Answer: a Explanation: 22nd National Conference on e-Governance 2019 successfully concluded at Shillong with the adoption of ‘Shillong Declaration’ on e-Governance. The Department of Administrative Reforms & Public Grievances (DAPRG), Ministry of Personnel and Ministry of Electronics & Information Technology (MeitY), in collaboration with the Government of Meghalaya organized the 22nd National Conference on eGovernance at Shillong. The theme of this Conference was “Digital India: Success to Excellence”. The Conference has unanimously adopted the ‘Shillong Declaration’ on e-Governance. Hence, option (a) is the correct answer. Additional Information: Key highlights of Shillong declaration:

• Improve the citizen’s experience with Government services by promoting timely implementation of India Enterprise Architecture (IndEA). • Consolidate the plethora of successful State level e-Governance projects with a focus to replicate them as a common application software. • Take steps to further improve connectivity in North Eastern States by addressing the issues of telecommunications connectivity at grassroot level. • Take steps to enhance the activities of Electronics Sector Skill Council in North Eastern States and explore the possibility for opening an electronics skill center in Shillong. • Promote use of e-Office and move towards less paper State Secretariats in the North- Eastern States and in the District level offices. Source: https://vajiramias.com/current-affairs/shillong-declaration-on-e- governance/5d4e710f1d5def64dbdb1554/ Q29. Which of the following pairs is/are correctly matched?

Dams Rivers

1. Sardar Sarovar Dam Narmada River 2. Hirakud Dam Mahanadi River

3. Mettur Dam Krishna River

Select the correct answer using the code given below: a. 1 only b. 1 and 2 only c. 2 and 3 only d. 1, 2 and 3 Answer: b Explanation: Pair (1) is correctly matched: The Sardar Sarovar Dam is a gravity dam on the Narmada river near Navagam, Gujarat. Power is shared among Madhya Pradesh, Maharashtra and Gujarat in a 57:27:16 ratio. Pair (2) is correctly matched: Hirakud Dam is built across the Mahanadi River in the state of Odisha. It is one of the first major multipurpose river valley projects started after India's independence. Pair (3) is not correctly matched: The Mettur Dam is one of the largest dams in India and the largest in Tamil Nadu, located across the river Cauvery where it enters the plains. Source: https://vajiramias.com/current-affairs/sardar-sarovar- project/5d4f90511d5def64da955ab0/ Q30. Copernicus Climate Change Service (C3S) is an initiative of: a. North Atlantic Treaty Organization (NATO) b. European Union c. National Aeronautics and Space Administration (NASA) d. African Union Answer: b Explanation: Copernicus is the European Union’s Earth Observation Programme. It consists of a complex set of systems which collect data from multiple sources: earth observation satellites and in situ sensors such as ground stations, airborne and sea borne sensors. The Copernicus Climate Change Service (C3S) supports society by providing authoritative information about the past, present and future climate in Europe and the rest of the World. The C3S mission is to support adaptation and mitigation policies of the European Union by providing consistent and authoritative information about climate change. Hence option (b) is the correct answer. Source: https://vajiramias.com/current-affairs/climate- change/5d510dc81d5def64e31c351d/ Q31. Which of the following are the sources of lead poisoning? 1. Polyvinyl Chloride (PVC) pipes 2. Paints 3. Toys and Jewelry 4. Pencils Select the correct answer using the code given below: a. 1 and 4 only b. 2 and 4 only c. 1, 2 and 3 only d. 1, 2, 3 and 4 Answer: c Explanation: (1) is correct: The National Green Tribunal (NGT) has recently directed the Centre to submit a report on standards of lead and to phase out lead as a stabiliser in Polyvinyl Chloride (PVC) pipes that are commonly used in most buildings. The NGT was earlier informed that water passing through PVC pipes commonly used in most buildings may contain toxic materials like lead, which has a hazardous effect on human health. While most of the organised manufacturers have started using alternative stabilisers such as zinc in PVC pipe-making, some of the small units still rely on lead due to the cost factor. The replacement of lead with zinc for stabilising heat in PVC pipe manufacturing is likely to push up costs by 3-5 per cent. (2) is correct: Lead is used in paint to add color, improve the ability of the paint to hide the surface it covers, and to make it last longer. (3) is correct: Lead has been found in inexpensive children's jewelry sold in vending machines and large volume discount stores across the country. It also has been found in inexpensive metal amulets worn for good luck or protection. (4) is not correct: There is no lead in pencils. Rather, the core is made up of a non-toxic mineral called graphite. Source: https://vajiramias.com/current-affairs/lead-in-pvc- pipes/5d5262261d5def64e1cc687e/ Q32. What is Perseid, recently seen in the news? a. Meteor Shower b. An Exoplanet c. Most distant object ever explored d. An 'X-Shaped' Galaxy Answer: a Explanation: The Perseids are one of the brighter meteor showers of the year. They occur every year between July 17 and August 24 and tend to peak around August 9-13. Made of tiny space debris from the comet Swift-Tuttle, the Perseids are named after the constellation Perseus. This is because the direction, or radiant, from which the shower seems to come in the sky lies in the same direction as Perseus. The Perseids are widely sought after by astronomers and stargazers because most years at its peak, one can see 60 to 100 meteors in an hour from a dark place. They are visible in the Northern Hemisphere and can be viewed in skies all across. Perseids are also known for their fireballs. Fireballs are larger explosions of light and color that can persist longer than an average meteor streak. Hence, option (a) is the correct answer. Source: https://vajiramias.com/current-affairs/perseids-meteor- shower/5d5262c41d5def64dfd9213d/ Q33. Which of the following initiatives have been taken by the Government of India to serve the differently-abled community of the country? 1. The Rights of Persons with Disabilities Act, 2016 2. National Handicapped Finance and Development Corporation (NHFDC) 3. Sugamya Bharat Abhiyan 4. Ratification of United Nations Convention on the Rights of Persons with Disabilities (UNCRPD) Select the correct answer using the code given below: a. 1 and 2 only b. 1, 2 and 3 only c. 1, 2 and 4 only d. 1, 2, 3 and 4 Answer: d Explanation: (1) is correct: The Union Cabinet has recently approved the proposal for Creation of two posts of Commissioners in the Office of the Chief Commissioner for Persons with Disabilities (CCPD). The recent decision has been taken in terms of Section 74(2) of The Rights of Persons with Disabilities (RPwD) Act, 2016. Out of the two posts of Commissioners, one Commissioner will be a person with disability. (2) is correct: National Handicapped Finance and Development Corporation (NHFDC) was set up by the Department of Disability Affairs in 1997 with the objective of serving as a catalyst in the economic development of Persons with Disabilities (PwDs) by helping them in setting up self-employment projects. (3) is correct: Accessible India Campaign or Sugamya Bharat Abhiyan is a program which is set to be launched to serve the differently-able community of the country. (4) is correct: India signed the United Nations Convention on the Rights of Persons with Disabilities (UNCRPD) and subsequently ratified the same on 1st October, 2007. The Rights of Persons with Disabilities Act, 2016 replaces the Persons with Disabilities (Equal Opportunities, Protection of Rights and Full Participation) Act, 1995 and is in line with the principles of the United Nations Convention on the Rights of Persons with Disabilities. Source: https://vajiramias.com/current-affairs/accessibility-standards-for-persons-with- disabilities-in-tv-programmes/5d55021b1d5def5610cdc819/ Q34. Which of the following best describes “CORAS”, recently seen in the news? a. A separate Commando Unit of Railway Protection Force (RPF). b. An international organization set up to promote democracy. c. An active composite volcano on the east coast of Sicily, Italy. d. A Nano-satellite designed and developed indigenously by Sri Lanka. Answer: a Explanation: CORAS is a separate Commando Unit of Railway Protection Force (RPF). These CORAS commandos will be posted in Left extremist wing affected areas, north-eastern region and Jammu and Kashmir where providing security to the passengers and the railway network is of utmost priority. A new commando training centre will be set up in Jagadhri, Haryana to modernize and train commandos of the Railway Protection Force (RPF). Hence, option (a) is the correct answer. Source: https://vajiramias.com/current-affairs/commandos-for-railway-security- coras/5d54ffe81d5def56157a554a/ Q35. With reference to the Gallantry Awards, consider the following statements: 1. There are only three gallantry awards instituted by the Government on 26th January, 1947. 2. These awards are announced twice in a year, i.e. Republic Day and Independence Day. Which of the statement/s given above is/are correct? a. 1 only b. 2 only c. Both 1 and 2 d. Neither 1 nor 2 Answer: b Explanation: Statement 1 is not correct: Three gallantry awards namely ParamVir Chakra, MahaVir Chakra and Vir Chakra were instituted by the Government of India on 26th January, 1950. Thereafter, other three gallantry awards i.e. the Ashoka Chakra Class-I, the Ashoka Chakra Class-II and the Ashoka Chakra Class-III were instituted in 1952. These awards were renamed as Ashoka Chakra, Kirti Chakra and Shaurya Chakra respectively in 1967. Statement 2 is correct: These gallantry awards are announced twice in a year - first on the occasion of the Republic Day and then on the occasion of the Independence Day. Order of precedence: Order of precedence of these awards is the ParamVir Chakra, the Ashoka Chakra, the Mahavir Chakra, the Kirti Chakra, the Vir Chakra and the Shaurya Chakra. Value Addition Tatrakshak Medals: Tatrakshak Medals are being given to the personnel of Indian Coast Guard on Republic Day and Independence Day every year, since January 26, 1990 onwards. Source: https://vajiramias.com/current-affairs/gallantry- awards/5d550a311d5def561495aba4/ Q36. Which of the following are the examples of terrestrial disasters? 1. Landslides 2. Tropical cyclone 3. Earthquake 4. Tsunami Select the correct answer using the code given below: a. 1 and 3 only b. 2 and 4 only c. 1, 3 and 4 only d. 1, 2 and 3 only Answer: a Explanation: Natural Disasters are broadly categorized as −Atmospheric Disasters, Terrestrial Disasters, Aquatic Disasters and Biological Disasters. Atmospheric disasters include blizzard, thunderstorm, lightning, tropical cyclone, tornado, drought, hailstorm, frost, heat wave, cold waves, etc. Terrestrial disasters include earthquake, volcanic eruption, landslide, avalanches, subsidence, etc. Aquatic disasters include flood, tidal waves, storm surge, tsunami, etc. Biological disasters include fungal, bacterial, and viral diseases (e.g. bird flu, dengue, etc.). Hence, option (a) is the correct answer. Source: https://vajiramias.com/current-affairs/kavalappara- landslide/5d564d111d5def682805f8c8/ Q37. With reference to the Maharshi Badrayan Vyas Samman, consider the following statements: 1. The awards cannot be given posthumously and to scholars convicted in criminal cases. 2. It is conferred based on work in the field of languages recognized in the Eighth Schedule of the Constitution. Which of the statement/s given above is/are correct? a. 1 only b. 2 only c. Both 1 and 2 d. Neither 1 nor 2 Answer: a Explanation: Maharshi Badrayan Vyas Sammanwere instituted in 1958 by Union Ministry of Human Resource Development. These are annually conferred on the Independence Day. It is conferred to recognise contribution of scholars the field of 9 languages (Sanskrit, Persian, Arabic, Pali, Prakrit, Classical Oriya, Classical Kannada, Classical Telugu and Classical Malayalam). These awards are not given posthumously. Also, these awards are not given to scholars -who have received this award earlier, or who are convicted in a criminal case/against whom a criminal case is pending in a court. Hence only statement 1 is correct. Source: https://vajiramias.com/current-affairs/maharshi-badrayan-vyas- samman/5d564fb61d5def68264c0e89/ Q38. Which of the following woodworks have been recognized with Geographical Indication (GI) tag in India? 1.Kondapalli Toys, Andhra Pradesh 2.Channapatna Toys, Karnataka 3. Wooden Dolls of Natungram, West Bengal Select the correct answer using the code given below: a. 2 only b. 1 and 2 only c. 2 and 3 only d. 1, 2 and 3 Answer: d Explanation: (1) is correct: Kondapalli Toys are the toys made of wood in Kondapalli village of Krishna district, Andhra Pradesh. It was registered as one of the geographical indication handicraft from Andhra Pradesh as per Geographical Indications of Goods (Registration and Protection) Act, 1999. The artisans mainly work on producing figures of mythology, animals, birds, bullock carts, rural life etc., and the most notable one is Dasavataram, dancing dolls etc. The artisans who make the toys are referred as Aryakhastriyas (also known as Nakarshalu), who have their mention in the Brahmanda Purana. The Kondapalli toys are made from soft wood known as TellaPoniki which are found in nearby Kondapalli Hills. (2) is correct: Channapatna toys are a particular form of wooden toys (and dolls) that are manufactured in the town of Channapatna in the Ramanagara district of Karnataka state, India. This traditional craft is protected under geographical indication (GI) tag. Though traditionally these toys were made of ivory, now they are made of sycamore, cedar, pine, teak and even rubber. These colourful toys are crafted carefully and indivdually, complete with vegetable dyes. (3) is correct: Natungram is the hub of the wooden doll makers in West Bengal. Carved out of a single piece of wood, these dolls from ancient folklore and mythology are characterised by their vibrant colour and ethnic style. Both men and women play important roles in the making of these dolls. The owl, considered auspicious as the escort of the goddess of prosperity, Lakshmi, made by Natungram enjoys an iconic status among West Bengal's handicraft. This traditional craft is also protected under geographical indication (GI) tag. Hence, option (d) is the correct answer. Source: https://vajiramias.com/current-affairs/kondapalli- toys/5d57aa0f1d5def68264c3dfa/ Q39. Jabro Dance and Spawo dance, are traditional dance forms prevailing in: a. Ladakh b. Tawang c. Gangtok d. Kohima Answer: a Explanation: Jabro Dance is a nomadic dance and song of people inhabiting eastern Ladakh and Spawo dance is a heroic song and dance associated with a legendary hero of Himalayan region called K’sar. Jabro dance is performed by men & Women at all festive occasions, this dance is an integral part of the Losar celebrations, which is the Tibetan New Year Festival. Hence, option (a) is the correct answer. Source: https://vajiramias.com/current-affairs/aadi- mahotsav/5d57acae1d5def68264c3e32/ Q40. With respect to “Know India Programme”, consider the following statements: 1. Its objective is to make Indian origin youth aware about India and its cultural heritage. 2. It gives preference to the PIOs from Girmitiya countries. 3. It is implemented by the Union Ministry of Home Affairs. Which of the above given statements is/are correct? a. 1 and 2 only b. 2 and 3 only c. 1 and 3 only d. 1, 2 and 3 Answer: a Explanation: Statement 1 is correct. Know India Programme (KIP) is a flagship initiative for Diaspora engagement which familiarizes Indian-origin youth (18-30 years) with their Indian roots and contemporary India. Statement 2 is correct. In 2016, the scheme was revamped to increase duration from21 to 25 days, with a 10- day visit to one or two States and preference given to PIOs from Girmitiya countries. “Girmitiyas” or Indentured Laborers, is the name given the Indians who left Indian in the middle and late 19th Century to serve as laborers in the British colonies, where the majority eventually settled. Mauritius, Fiji, Suriname, Guyana, Trinidad & Tobago are known as Girmitiya Countries. Statement 3 is not correct. It is organised by the Ministry of External Affairs. Source: https://vajiramias.com/current-affairs/know-india- programme/5d58e4331d5def682730756f/ Q41. Which of the following pairs is/are correctly matched?

Spacecraft Purpose

1. Juno Explore Jupiter's formation and evolution

2. InSight Exploring the outer solar system

3. Voyager 1and 2 Study in-depth the "inner space" of Mars: its crust, mantle, and core.

Select the correct answer using the code given below: a. 1 only b. 1 and 2 only c. 2 and 3 only d. 1, 2 and 3 Answer: a Explanation: (1) is correct: Juno is a NASA space probe orbiting the planet Jupiter. The spacecraft was launched in 2011 as part of the New Frontiers program, and entered a polar orbit of Jupiter in 2016, to begin a scientific investigation of the planet. Juno’s mission is to measure Jupiter’s composition, gravity field, magnetic field, and polar magnetosphere. Juno is the second spacecraft to orbit Jupiter, after the nuclear powered Galileo orbiter, which orbited from 1995 to 2003. Unlike all earlier spacecraft sent to the outer planets, Juno is powered by solar arrays. (2) is not correct: InSight, short for Interior Exploration using Seismic Investigations, Geodesy and Heat Transport, is a Mars lander designed to give the Red Planet its first thorough checkup since it formed 4 billion years ago. It is the first outer space robotic explorer to study in-depth the "inner space" of Mars: its crust, mantle, and core. (3) is not correct: Voyager 1 reached interstellar space in August 2012 and is the most distant human-made object in existence. Launched just shortly after its twin spacecraft, Voyager 2, in 1977, Voyager 1 explored the Jovian and Saturnian systems discovering new moons, active volcanoes and a wealth of data about the outer solar system. It is part of the Voyager program to study the outer Solar System. Voyagers 1 and 2 were designed to take advantage of a rare planetary alignment that occurs only once in 176 years and remain the most well-traveled spacecraft in history. Source: https://vajiramias.com/current-affairs/juno/5d5a4af91d5def3e51ab0b91/ Q42. With reference to Bond Yield curve, consider the following statements: 1. The steepness of this yield curve is determined by how fast an economy is expected to grow. 2. Yield inversion happens when the yield on a longer tenure bond becomes less than the yield for a shorter tenure bond. Which of the statements given above is/are correct? a. 1 only b. 2 only c. Both 1 and 2 d. Neither 1 nor 2 Answer: c Explanation: A bond is an instrument to borrow money. A bond could be floated/issued by a country’s government or by a company to raise funds.A yield curve is a graphical representation of yields for bonds (with an equal credit rating) overdifferent time horizons. If bond investors expect the economy to grow normally, then they would expect to be rewarded more (that is, get more yield) when they lend for a longer period. This gives rise to a normal —upward sloping — yield curve. The steepness of this yield curve is determined by how fast an economy is expected to grow. When the economy is expected to grow only marginally, the yield curve is “flat”. Yield inversion happens when the yield on a longer tenure bond becomes less than the yield for a shorter tenure bond. A yield inversion typically portends a recession. An inverted yield curve shows that investors expect the future growth to fall sharply. Hence both statements are correct. Source: https://vajiramias.com/current-affairs/bonds-yield- curve/5d5b95001d5def42939a26f2/ Q43. Which of the following are the initiatives or joint effort of the NITI Aayog? 1. State Rooftop Solar Attractiveness Index (SARAL) 2. Agricultural Marketing and Farm Friendly Reforms Index 3. Women Transforming India (WTI) Awards 2019 Select the correct answer using the code given below: a. 1 and 3 only b. 2 and 3 only c. 1 and 2 only d. 1, 2 and 3 only Answer: b Explanation: (1) is not correct: Union Power Minister has recently launched the State Rooftop Solar Attractiveness Index (SARAL) during the Review Planning and Monitoring (RPM) Meeting with States and State Power Utilities. Objective of Index is to incentivise rooftop solar by creating healthy competition among the States. SARAL has been designed collaboratively by the Ministry of New and Renewable Energy (MNRE), Sustainable Energy Foundation (SSEF), Associated Chambers of Commerce and Industry of India (ASSOCHAM) and Ernst & Young (EY). The Index evaluates Indian states based on their attractiveness for rooftop development. SARAL currently captures five key aspects – Robustness of policy framework, Implementation environment, Investment climate, Consumer experience and Business ecosystem. Karnataka has been placed at the first rank in the Index. Telangana, Gujarat and Andhra Pradesh have got 2nd, 3rd and 4th rank respectively. (2) is correct: NITI Aayog brings out an Agricultural Marketing and Farm Friendly Reforms Index, rating States on their implementation of such reforms. In the initial edition of that Index in 2016, Maharashtra stood first in the rankings, followed by Gujarat. Index is based on implementation of seven provisions proposed under model APMC Act like joining e- NAM initiative, special treatment to fruits and vegetables for marketing and level of taxes in mandis. (3) is correct: Women Transforming India (WTI) Awards 2019 is a flagship initiative of NITI Aayog. Recently NITI Aayog and Facebook-owned WhatsApp, under the aegis of its Women Entrepreneurship Platform (WEP), have announced a partnership to support the growth of a robust ecosystem to promote women entrepreneurs in India. The WTI Awards, since its inception in 2016, has been recognising and celebrating stories of exemplary women from across India. The theme for WTI Awards 2019 is “Women and Entrepreneurship”. Source: https://vajiramias.com/current-affairs/state-rooftop-solar-attractiveness-index- saral/5d5e1a671d5def50d3797f6b/ Q44. What is ‘Op-Blue Freedom’, recently seen in the news? a. It is a new Umbrella Multi-Donor Trust Fund (MDTF), housed at the World Bank that supports healthy and productive oceans. b. It is a nation-wide adaptive scuba diving programme for people with disabilities as well as the able bodied. c. It is an initiative of Government of India to develop seaports and maritime transport sectors under Sagarmalaprogramme. d. It is an initiative of WWF for the sustainable economic development of the ocean. Answer: b Explanation: Op-Blue Freedom is a nation-wide adaptive scuba diving programme for people with disabilities as well as the able bodied. This is an initiative in which a group of armed forces veterans train adventure enthusiasts in special forces skills like survival techniques, emergency first response skills, unarmed combat, endurance and fitness. Hence, option (b) is the correct answer. Addition Information: PROBLUE is a new Umbrella Multi-Donor Trust Fund (MDTF), housed at the World Bank that supports healthy and productive oceans. PROBLUE supports implementation of Sustainable Development Goal 14 (SDG 14) and is fully aligned with the World Bank’s twin goals of ending extreme poverty and increasing the income and welfare of the poor in a sustainable way. PROBLUE is part of the World Bank’s overall Blue Economy program, which takes a multi-pronged, coordinated approach to ensuring the protection and sustainable use of marine and coastal resources. Source: https://vajiramias.com/current-affairs/op-blue- freedom/5d5e29371d5def50de131b8c/ Q45. Which of the following have been declared as renewable energy sources in India? 1. Ocean energy projects 2. Solar projects 3. Wind power projects 4. Large hydropower projects Select the correct answer using the code given below. a. 1 and 2 only b. 2 and 3 only c. 1, 2 and 3 only d. 1, 2, 3 and 4 Answer: d Explanation: The New and Renewable Energy Ministry has clarified that energy produced using various forms of ocean energy such as tidal, wave and ocean thermal energy conversion will be considered as Renewable Energy. It will be eligible for meeting the non-solar Renewable Purchase Obligations (RPO). The decision will give a boost to ocean energy in the country. Ocean energy is mostly exploited following technologies –

• Tidal Energy: The tidal cycle occurs every 12 hours due to the gravitational force of the moon. The difference in water height from low tide and high tide is potential energy. • Wave Energy: Wave energy is generated by the movement of a device either floating on the surface of the ocean or moored to the ocean floor. • Current Energy: Marine current is ocean water moving in one direction. This ocean current is known as the Gulf Stream. Tides also create currents that flow in two directions. Kinetic energy can be captured from the Gulf Stream and other tidal currents with submerged turbines • Ocean Thermal Energy Conversion (OTEC): OTEC, uses ocean temperature differences from the surface to depths lower than 1,000 meters, to extract energy. A temperature difference of only 20°C can yield usable energy.

As part of the new hydro policy, large hydropower projects have been declared as renewable energy sources, making them a part of the renewable purchase obligation. Of the targeted 175GW of RE (renewable energy) capacity by 2022, 100GW is to come from solar projects. It will comprise 60GW from ground-mounted, grid-connected projects, and 40GW from solar rooftop projects. Wind power projects will contribute 60GW. Hence, option (d) is the correct answer. Additional Information: The renewable purchase obligation (RPO) mechanism is a market-based instrument to promote renewable sources of energy and development of market in electricity. It provides an alternative voluntary route to a generator to sell his electricity from renewable sources just like conventional electricity and offer the green attribute (RECs) separately to obligated entities to fulfil their RPO. Source: https://vajiramias.com/current-affairs/ocean-energy/5d5f85131d5def50d54aabfd/ Q46. With reference to the buyback shares, consider the following statements: 1. A buyback can be done only through tender offer route. 2. Buyback is usually done at a price higher than the then prevailing market price shareholders get an attractive exit option. Which of the statements given above is/are correct? a. 1 only b. 2 only c. Both 1 and 2 d. Neither 1 nor 2 Answer: b Explanation: Statement 1 is not correct: A buyback is a mechanism through which a listed company buys back shares from the market. A buyback can be done either through open market purchases or through the tender offer route. Under the open market mechanism, the company buys back the shares from the secondary market. Historically, most companies had preferred the open market route. Under tender offer, shareholders can tender their shares during the buyback offer. Statement 2 is correct: Buybacks are typically done when a company has a significant cash reserve and feels that the shares are not fairly valued at the current market price. Since the shares that are bought back are extinguished, the stake of the remaining shareholders rise. Since a buyback is usually done at a price higher than the then prevailing market price, shareholders get an attractive exit option, especially when the shares are thinly traded. It is also more tax- efficient than dividends as a way to reward shareholders. Promoters also use this mechanism to tighten their grip on the firm. Source: https://vajiramias.com/current-affairs/buyback/5d5f864b1d5def50d9349409/ Q47. What is ‘AkademikLomonosov’, recently seen in the news? a. Anti-tank missile developed by Russia. b. World’s first floating nuclear reactor launched by Russia. c. Composite volcano in the northern region of Siberia. d. World’s southernmost city. Answer: b Explanation: Russia recently launched the world’s first floating nuclear reactor named AkademikLomonosov. Hence, option (b) is the correct answer.It was named after 18th century Russian scientist Mikhail Lomonosov. It was developed by Rosatom nuclear agency. The floating vessel will be the northernmost nuclear plant in the world, and will also power the extraction of natural resources in the region. Source: https://vajiramias.com/current-affairs/akademik- lomonosov/5d6233611d5def7c5afc9ae4/ Q48. Which of the following pairs is/are correctly matched?

Indigenous people Country

1. Inuit Greenland

2. Basques Chile

3. Sami South Africa

Select the correct answer using the code given below. a. 1 only b. 1 and 2 only c. 2 and 3 only d. 1, 2 and 3 Answer: a Explanation: Pair (1) is correctly matched: Greenland, an autonomous province of Denmark, is located to the north east of Canada. Most of its inhabitants are Inuit whose ancestors migrated from Canada. The Inuit are a group of culturally similar indigenous peoples inhabiting Inuit Nunangat, the Arctic regions of Greenland, Canada, and Alaska. Pair (2) is not correctly matched: The Basques are a European indigenous ethnic group characterised by the Basque language, a common culture and shared genetic ancestry to the ancient Vascones and Aquitanians. They mainly live in both Spain and France in areas bordering the Bay of Biscay and encompassing the western foothills of the Pyrenees Mountains. Pair (3) is not correctly matched: Sami is an indigenous group of people speaking the Sami language and inhabiting Lapland and adjacent areas of northern Norway, Sweden, and Finland, as well as the Kola Peninsula of Russia. Source: https://vajiramias.com/current-affairs/greenland/5d62353c1d5def7c56dc351c/ Q49. Which of the following species have been included in the Appendix I of Convention on International Trade in Endangered Species of Wild Fauna and Flora (CITES) during the recent Eighteenth Conference of the Parties (CoP18)? 1. Smooth-coated otter 2. Tokay gecko 3. Mako sharks 4. Indian star tortoise Select the correct answer using the code given below: a. 1 and 4 only b. 2 and 3 only c. 1, 2 and 3 only d. 1, 2, 3 and 4 Answer: a Explanation: The Eighteenth Conference of the Parties (CoP18) of the Convention on International Trade in Endangered Species of Wild Fauna and Flora (CITES) was recently held in Geneva. (1) is correct: It was decided to move the smooth-coated otter (Lutrogaleperspicillata) from CITES Appendix II to CITES Appendix I. Smooth-coated otter numbers in the wild had fallen by at least 30% over the past 30 years. Smooth-coated and Asian small-clawed otters, both found in large numbers across India, especially along the Konkan coast. (2) is not correct: The other proposal that was passed was to include the Tokay gecko (Gekko gecko) in CITES Appendix II. The gecko is used in Chinese traditional medicine and traded throughout south-east Asia in dried form or preserved in alcohol. (3) is not correct: Mako sharks, giant guitarfish and wedgefish are among those which have now been included in Appendix II of CITES, enabling sustainable trade in them. The species included in the list are endangered shortfin and longfinmako shark, six species of giant guitarfish and 10 species of wedgefish. Of the giant guitarfish and wedgefish species on the listing, all except one of the wedgefish species are critically endangered. (4) is correct: The Indian star tortoise was upgraded to CITES Appendix I (threatened with extinction) by full consensus among all member states (of CoP18 with 183 countries), giving it the highest level of international protection from commercial trade. Additional Information: CITES Appendix:

• Appendix I includes species “threatened with extinction”. Trade in specimens of these species is permitted only in exceptional circumstances. • Appendix II provides a lower level of protection. • Appendix III contains species that are protected in at least one country, which has asked other CITES Parties for assistance in controlling the trade. Convention on International Trade in Endangered Species of Wild Fauna and Flora (CITES):

• The CITES is as an international agreement aimed at ensuring “that international trade in specimens of wild animals and plants does not threaten their survival”. • The text of the Convention was agreed in Washington, DC, in 1973 and entered into force in 1975. Thus, the convention is sometimes referred to as the Washington Convention. • The convention now has 183 parties. • The Convention is legally binding on the Parties in the sense that they are committed to implementing it; however, it does not take the place of national laws. Source: https://vajiramias.com/current-affairs/convention-on-international-trade-in- endangered-species-of-wild-fauna-and-flora-cites/5d64d4c41d5def0e82b996e4/ Q50. With reference to the “Ghar Ghar Dastak Ghar Ghar Pustak”, consider the following statements: 1. It is an initiative of Union Ministry of Human Resource Development. 2. Under this scheme, books will be provided to the residents of slums, resettlement colonies and rural areas in all over the country. Which of the statements given above is/are correct? a. 1 only b. 2 only c. Both 1 and 2 d. Neither 1 nor 2 Answer: d Explanation: The Union Minister of State for Culture and Tourism (I/C) recently launched the Mobile Library Busesprocured under C.S.R assistance from Airports Authority of India in New Delhi. The Programme was organized by Delhi Public Library of Ministry of Culture. Hence statement 1 is incorrect. The Mobile Library Buses were launched under the “GharGharDastakGharGharPustak”Scheme of the Delhi Public Library.Under this scheme, the Delhi Public Library aims to provide books to the citizens of Delhi, especially, the residents of slums, resettlement colonies and rural areas.Hence statement 2 is incorrect. Additional Information: Delhi Public Library

• Delhi Public Library is a national depository library in Delhi. • The Library was established in 1951 as a pilot project sponsored by UNESCO and theGovernment of India. • It functions under the administrative control of the Ministry of Culture, Government of India. Source: https://vajiramias.com/current-affairs/ghar-ghar-dastak-ghar-ghar- pustak/5d4297271d5def4ee8491cc9/ Q51. With reference to the 14th Conference of Parties (COP14) to the United Nations Convention to Combat Desertification (UNCCD), consider the following statements: 1. COP 14 to UNCCD adopted the Delhi Declaration which is a statement of global action by each country to achieve Land Degradation Neutrality (LDN) by 2030. 2. The Peace Forest Initiative (PFI) is a global initiative serving as a partnership framework with a focus on land degradation neutrality (LDN) in fragile and conflict-affected regions. Which of the statements given above is/are correct? a. 1 only b. 2 only c. Both 1 and 2 d. Neither 1 nor 2 Answer: c Explanation: The 14th Conference of Parties (COP14) to the United Nations Convention to Combat Desertification (UNCCD) recently held in India with member countries adopting the Delhi Declaration. COP 14 to UNCCD ended with a commitment to achieve Land Degradation Neutrality (LDN) by 2030. Such neutrality is defined by the UN as ensuring that enough land is available across the world to ensure a sustainable future. The Delhi Declaration is a statement of global action by each country on how to achieve Land Degradation Neutrality. The Peace Forest Initiative (PFI) is a global initiative serving as a partnership framework with a focus on land degradation neutrality (LDN) in fragile and conflict-affected regions. The overall objective is to promote peace and build trust between neighbouring countries through transboundary co-operation in sustainable land management, including forest. The initiative is designed to provide a practical platform that will foster international collaboration by demonstrating the value of achieving land degradation neutrality (LDN, SGD target 15.3) in cross-border post-conflict situations. Cooperative efforts on actions such as sustainable land management, forestry and land rehabilitation can facilitate economic cooperation, while supporting the achievement of multiple SDG targets. Hence both statements are correct. Delhi Declaration COP 14: Key Points 1. Promote projects to prevent land degradation, drought and erosion at local, national and regional levels. 2. Create green jobs by promoting technology and investment in collaboration with all stakeholders. 3. Implement the UN Ecosystem Restoration (2021-2030) program on behalf of all countries. 4. It takes note of the launch of the Peace Forest Initiative and its potential contribution to increasing cooperation on land degradation neutrality. 5. Encourage local governments to adopt integrated land use management and enhanced land governance to rehabilitate the natural resource base. 6. COP-14 member countries appreciate India's efforts. PM Modi's efforts to achieve the major goal of land restoration and increase south-south cooperation were praised. Source:https://vajiramias.com/current-affairs/delhi-declaration-unccd-cop14- concludes/5d7dc3661d5def0d29521dfa/ Q52. Consider the following statements: 1. It lies on the Coromandel Coast which faces the Bay of Bengal. 2. It is a well-established sea port was during the 7th to 10th centuries of the Pallava dynasty. 3. It is also known as Seven Pagodas. 4. The entire assemblage of monuments of this town collectively designated as a UNESCO World Heritage site in 1984. Above statements describes which of the following places? a. Mamallapuram b. Puri c. Kakinada d. Madurai Answer: a Explanation: Mamallapuram (Mahabalipuram) is a town in Chengalpattu district, Tamil Nadu. It lies on the Coromandel Coast which faces the Bay of Bengal.Mamallapuram was one of two major port cities by the 7th century within the Pallava kingdom.The town has a collection of 7th and 8th- century religious monuments that have been declared as a UNESCO World Heritage site. Some important structures include: Thirukadalmallai, Descent of the Ganges or Arjuna’s Penance, Varaha Cave Temple and Pancha Rathas (Five Chariots). Mamallapuramwas the venue for the second India-China informal summit. Hence, option (a) is the correct answer.

UNESCO Description of Mahabalipuram:

This group of sanctuaries, founded by the Pallava kings, was carved out of rock along the Coromandel coast in the 7th and 8th centuries. It is known especially for its rathas (temples in the form of chariots), mandapas (cave sanctuaries), giant open-air reliefs such as the famous 'Descent of the Ganges', and the temple of Rivage, with thousands of sculptures to the glory of .

Source:https://vajiramias.com/current- affairs/mamallapuram/5d6ccd931d5def56141b3587/ Q53. Which of the following is the largest island in Asia? a. Sumatra b. Honshu c. Java d. Borneo Answer: d Explanation: Borneo is the third-largest island in the world and the largest in Asia. The island is politically divided among three countries: Malaysia, Brunei and Indonesia. Kalimantan is the Indonesian portion of the island of Borneo. It comprises 73% of the islands area.Indonesia’s president, JokoWidodo has recently announced the national capital will move from Jakarta, on the island of Java, to the province of East Kalimantan, on Borneo.Hence, option (d) is the correct answer.

Source:https://vajiramias.com/current-affairs/kalimantan/5d6b6d2b1d5def4d216e2eaa/ Q54. Which of the following industries comes under the Eight Core Industries? 1. Coal 2. Steel 3. Cement Select the correct answer using the code given below: a. 1 and 3 only b. 1 and 2 only c. 2 and 3 only d. 1, 2 and 3 Answer: d Explanation: The eight core industries of coal, crude oil, natural gas, refinery products, fertilisers, steel, cement and electricity, which have a 40.27% weight in the Index of Industrial Production. The Index of Eight Core Industries is a monthly production index, which is also considered as a lead indicator of the monthly industrial performance. Hence, option (d) is the correct answer.

The Index of Industrial Production (IIP) is an index for India which details out the growth of various sectors in an economy such as mineral mining, electricity and manufacturing. The all India IIP is a composite indicator that measures the short-term changes in the volume of production of a basket of industrial products during a given period with respect to that in a chosen base period. It is compiled and published monthly by the Central Statistics Office (CSO). The Eight Core Industries comprise nearly 40.27% of the weight of items included in the Index of Industrial Production (IIP).

Source:https://vajiramias.com/current-affairs/growth-of-eight-core- sectors/5d6e16b81d5def5e405c0c0c/ Q55. ‘Nine-dash line’ often mentioned in news, is in: a. South China Sea b. Bay of Bengal c. Arabian Sea d. Mediterranean Sea Answer: a Explanation: Nine-dash line is in the South China Sea. The ‘nine-dash line’ stretches hundreds of kilometres south and east of its southerly Hainan Island, covering the strategic Paracel and Spratly island chains. China claims it by citing 2,000 years of history when the two island chains were regarded as its integral parts. There are a few hundred small islands in the SCS, a part of the Pacific Ocean. Some of the main ones are the Spratly Islands, Paracel Islands and Scarborough Shoal — the bone of contention between China and the Philippines. China claims most of these islands as its own. Vietnam, Malaysia, Brunei, the Philippines and Taiwan have rival claims. Hence, option (a) is the correct answer.

Source:https://vajiramias.com/current-affairs/asean-us-maritime-exercise- aumx/5d6e15c71d5def5e3719e4f0/ Q56. With reference to the Global Tiger Forum (GTF), consider the following statements: 1. It is the only intergovernmental international body established to protect the remaining sub- species of Tigers distributed over 13 Tiger Range countries of the world. 2. It was formed in 1993 and the secretariat is located in Moscow. Which of the statements given above is/are correct? a. 1 only b. 2 only c. Both 1 and 2 d. Neither 1 nor 2 Answer: a Explanation: Statement 1 is correct: Global Tiger Forum (GTF) is the only intergovernmental international body established with members from willing countries to embark on a global campaign to protect the remaining sub- species of Tigers distributed over 13 Tiger Range countries of the world. Statement 2 is incorrect: The GTF was formed in 1993. In 1997, the GTF became an independent organization.The GTF has a General Assembly meeting every 3 years and Standing committee meetings at least once a year. A Chairperson, usually a Minister from one of the Tiger Rangecountries heads GTF for a fixed tenure of 3 Years.Secretariat is at New Delhi. Tiger sub-species –

Since 2017, IUCN has recognized two tiger subspecies, commonly referred to as the continental tiger and the Sunda island tiger. All remaining island tigers are found only in Sumatra, with tigers in Java and Bali now extinct. These are popularly known as Sumatran tigers. The continental tigers currently include the Bengal, Malayan, Indochinese and Amur (Siberian) tiger populations, while the Caspian tiger is extinct in the wild. The South China tiger is believed to be functionally extinct.

Source:https://vajiramias.com/current-affairs/tigers-at-high- altitude/5d6f69a61d5def5e3d11890f/ Q57. Asteroid Impact Deflection Assessment (AIDA), often mentioned in news, is an initiative of: a. National Aeronautics and Space Administration (NASA) b. European Space Agency (ESA) c. Indian Space Research Organization (ISRO) d. Both a and b Answer: d Explanation: Asteroid Impact Deflection Assessment (AIDA) is a joint research mission between NASA and the European Space Agency (ESA) teams.It aims to study the viability of diverting an asteroid by crashing a spacecraft into its surface.The project aims to deflect the orbit of one of the two Didymos asteroids between Earth and Mars, with an observer craft gauging the effect of the impact more effectively than ground-based observers could manage.Hence, option (d) is the correct answer.

Asteroids are larger rocks that orbit around the sun. Often, due to gravitational pulls from planets, they can be moved off course and can collide with planets and other debris in space. A meteor is a smaller rock that flies around our solar system.

Didymos is a binary asteroid; the primary body has a diameter of around 780 m and a rotation period of 2.26 hours, whereas the Didymoon secondary body has a diameter of around 160 m and rotates around the primary at a distance of around 1.2 km from the primary surface in around 12 hours.

Source:https://vajiramias.com/current-affairs/asteroid-impact-deflection-assessment- aida/5d6f668c1d5def5e371a19aa/ Q58. Which of the following pairs is/are correctly matched?

Festivals States

1. Nuakhai Odisha

2. Bihu West Bengal

3. Onam Tamil Nadu

Select the correct answer using the code given below: a. 1 only b. 1 and 2 only c. 2 and 3 only d. 1, 2 and 3 Answer: a Explanation: Pair (1) is correctly matched:‘Nuakhai’is an important social festival of Western Odisha and adjoining areas of Simdega in Jharkhand.It is an agricultural festival which is observed to welcome the new rice of the season.The word nua means new and khai means food.It is observed on the fifth day of the lunar fortnight of the month of Bhadrapada or Bhaadra (August– September), the day after the Ganesh Chaturthi festival.People offer the newly harvested crop called Nabanha to their respective presiding deities. After offering the Nabanha, people relish different types of delicacies prepared from the newly harvested crop. The NuakhaiJuhar is a major ritual of the festival. It is an exchange of greetings with friend relative and well-wishers. Pair (2) is not correctly matched:Bihu is a set of three important non-religious festivals in the Indian state of Assam—Rongali or BohagBihu observed in April, Kongali or Kati Bihu observed in October, and Bhogali or Magh Bihu observed in January. The RongaliBihu is the most important of the three, celebrating the spring festival. Pair (3) is not correctly matched:Onam is an annual Harvest festival in the state of Kerala in India. It is a traditional ten-day harvest festival that marks the homecoming of the mythical King Mahabali. Source:https://vajiramias.com/current-affairs/nuakhai/5d709b291d5def5e39292560/ Q59. With reference to the Eastern Economic Forum, consider the following statements: 1. It was established by the United Nations Security Council in 2015. 2. It aims to support the economic development of Russia’s Far East and to expand international cooperation in the Asia-Pacific region. Which of the statements given above is/are correct? a. 1 only b. 2 only c. Both 1 and 2 d. Neither 1 nor 2 Answer: b Explanation: The Eastern Economic Forum was established by decree of the President of the Russian Federation Vladimir Putin in 2015. It aims to support the economic development of Russia’s Far East and to expand international cooperation in the Asia-Pacific region. Events at the Forum traditionally take place in the form of panel sessions, roundtables,televised debates, business breakfasts, and business dialogues devoted to Russia’s relationships with various countries.Hence only statement 2 is correct. Sourcehttps://vajiramias.com/current-affairs/eastern-economic- forum/5d709d541d5def5e405c75d1/ Q60. With reference to Rashtriya Bal Swasthya Karyakram (RBSK), consider the following statements: 1. It was launched in 2018 under the Ayushman Bharat Yojana. 2. This initiative is aimed at screening children from 0 to 18 years for 4Ds - Defects at birth, Diseases, Deficiencies and Development Delays including Disabilities. Which of the statements given above is/are correct? a. 1 only b. 2 only c. Both 1 and 2 d. Neither 1 nor 2 Answer: b Explanation: Statement 1 is incorrect:

Rashtriya Bal Swasthya Karyakram (RBSK) was launched in 2013 under the National Health Mission (NHM), RBSK is focused on preventing disease and disability in children. Statement 2 is correct: This initiative is aimed at screening children from 0 to 18 years for 4Ds - Defects at birth, Diseases, Deficiencies and Development Delays including Disabilities.Children diagnosed with illnesses shall receive follow up including surgeries at tertiary level, free of cost under NHM. Source:https://vajiramias.com/current-affairs/rashtriya-bal-swasthya-karyakram- rbsk/5d709c711d5def5e3929258b/ Q61. The Reserve Bank of India (RBI) has made it mandatory for banks to link all of their new loan products to an external benchmark. Which of the following is qualified to be an external benchmark? 1. Repo rate 2. 3-month or 6-month treasury bill yield 3. The marginal cost of funds-based lending rate (MCLR) 4. Any other benchmark published by the Financial Benchmarks India Pvt. Ltd. Select the correct answer using the code given below: a. 1 and 3 only b. 2 and 3 only c. 1, 2 and 4 only d. 1, 2, 3 and 4 Answer: c Explanation: The Reserve Bank of India (RBI) has made it mandatory for banks to link all of their new loan products, be it personal, housing or auto to an external benchmark like the policy repo rate. Banks can also choose any benchmark market interest rate published by Financial Benchmarks India Private Limited (FBIL) or the government’s 3-month and 6-month Treasury bill yields published by FBIL as their preferred external benchmark.The discussion was kicked off in August 2017, when RBI constituted an Internal Study Group (ISG) to examine the efficacy of the MCLR system, which reported that the system did not allow for effective transmission of rate cuts to customers. In light of this finding, RBI decided to mandate a switch-over to an external benchmark for lending rates, as recommended by the ISG. The repo (or repurchase) rate is the rate at which the Reserve Bank of India (RBI) lends money to other banks. Hence, cuts in the repo rate are meant to lead to cuts in the home loan and other lending rates as banks get to borrow money cheaply from the RBI. MCLR is a tenor-linked internal benchmark, which means the rate is determined internally by the bank depending on the period left for the repayment of a loan. MCLR is closely linked to the actual deposit rates and is calculated based on four components: the marginal cost of funds, negative carry on account of cash reserve ratio, operating costs and tenor premium. Hence, option (c) is the correct answer. Source:https://vajiramias.com/current-affairs/external-benchmark-based- lending/5d7200b91d5def3918ca66e5/ Q62. With reference to the Global Liveability Index, 2019, consider the following statements: 1. New York has been ranked the ‘world’s most liveable city’. 2. It is released by the World Economic Forum (WEF). Which of the statements given above is/are correct? a. 1 only b. 2 only c. Both 1 and 2 d. Neither 1 nor 2 Answer: d Explanation: The Austrian capital Vienna has been ranked the ‘world’s most liveable city’ for the second year in a row on the annual Global Liveability Index.Hence statement 1 is incorrect. The annual Global Liveability Index of 140 cities around the world, is released by the Economist Intelligence Unit (EIU), the research and analysis division of The Economist Group which brings out the UK’s The Economist news magazine.The index considers more than 30 qualitative and quantitative factors spanning categories like stability, healthcare, culture and environment, education, and infrastructure. For the first time,the index noted the effects of climate change on liveability.Scores are compiled on a scale of 1-100.Hence statement 2 is incorrect. 2019 Rankings:

• Vienna topped the rankings followed by Melbourne, Sydney, Osaka and Calgary. • New Delhi was ranked at 118th (a drop of six ranks) due to “poor air quality. Mumbai was ranked 119th. • The least liveable cities were Karachi, Tripoli, Dhaka, Lagos and, at the very bottom, Damascus(Syria). • Western Europe and North America continue to be the most liveable regions in the world. Source: https://vajiramias.com/current-affairs/global-liveability-index- 2019/5d7200391d5def391b986c86/ Q63. Which of the following are the benefits of getting Institutions of Eminence (IoE) tag? 1. Government Institutions to get additional funding up to 1000 Crore. 2. Academic collaborations with foreign higher educational institutions (in top 500) would be exempt from government approvals. 3. UGC Inspection shall not apply to Institutions of Eminence. Select the correct answer using the code given below: a. 1 only b. 1 and 2 only c. 2 and 3 only d. 1, 2 and 3 Answer: d Explanation: The institutes of eminence scheme under the Union ministry of Human Resource Development (HRD) aims to project Indian institutes to global recognition.The selected institutes will enjoy complete academic and administrative autonomy. Benefits of Institutions of Eminence

• Government Institutions to get additional funding up to 1000 Crore. • The selected Institutions under IoE shall have complete academic, administrative and financial autonomy. • Academic collaborations with foreign higher educational institutions (in top 500) would be exempt from government approvals; Freedom to recruit faculty from outside India (limit of 25% of its faculty strength for a public institution). • Freedom to admit additionally foreign students on merit subject to a maximum of 30% of the strength of admitted domestic students. Freedom to fix and charge fees from foreign students without restriction. • Freedom to have own transparent merit-based system for admission of students. • Flexibility in fixing of curriculum and syllabus, with no UGC mandated curriculum structure. • Freedom to offer online courses as part of their programmes with a restriction that not more than 20% of the programme should be in online mode. • UGC Inspection shall not apply to Institutions of Eminence. Hence, option (d) is the correct answer. Source:https://vajiramias.com/current-affairs/institution-of-eminence- ioe/5d7202881d5def39107220a0/ Q64. Which of the following diseases are caused by viruses? 1. Measles 2. Rubella 3. Chikungunya 4. Tetanus Select the correct answer using the code given below: a. 1 and 3 only b. 1 and 4 only c. 2, 3 and 4 only d. 1, 2 and 3 only Answer: d Explanation: Point (1) is correct: Measles is a highly contagious viral disease. It is transmitted via droplets from the nose, mouth or throat of infected persons. Point (2) is correct: Rubella, also called German measles or three-day measles, is a contagious viral infection best known by its distinctive red rash. Rubella is not the same as measles (rubeola), though the two illnesses do share some characteristics, including the red rash. Rubella infection in pregnant women may have serious consequences causing miscarriages, stillbirths or severe birth defects known as congenital rubella syndrome (CRS). Point (3) is correct: Chikungunya is a viral disease (genus Alphavirus) which is transmitted to humans by infected mosquitoes, most commonly, Aedesaegypti and Aedesalbopictus, two species which can also transmit other mosquito-borne viruses, including dengue. These mosquitoes can be found biting throughout daylight hours,though there may be peaks of activity in the early morning and late afternoon. It causes fever and severe joint pain. The disease shares some clinical signs with dengue and Zika, and can be misdiagnosed in areas where they are common. Point (4) is incorrect: Tetanus, also called lockjaw, is a serious infection caused by Clostridium tetani. This bacterium produces a toxin that affects the brain and nervous system, leading to stiffness in the muscles. Hence, option (d) is the correct answer. Source:https://vajiramias.com/current-affairs/measles- rubella/5d7345bb1d5def43be3fa969/ Q65. India recently signed an MoU to open a full-fledged maritime route between Chennai on India’s eastern seaboard and which of the following port city? a. Vladivostok, eastern Russia b. Murmansk, northwest Russia c. Vysotsk, northern Russia d. Port of Haydarpasa, Turkey Answer: a Explanation: During Prime Minister of India’s recent visit to Vladivostok, Russia, an MoU was signed to open a full-fledged maritime route between Russia’s eastern port city and Chennai on India’s eastern seaboard. Located on the Golden Horn Bay north of North Korea and a short distance from Russia’s border with China, Vladivostok is the largest port on Russia’s Pacific coast, and home to the Pacific Fleet of the Russian Navy.At Vladivostok’s massive port, shipping and commercial fishing are the main commercial activities.Hence, option (a) is the correct answer. An ocean liner travelling from Vladivostok to Chennai would sail southward on the Sea of Japan past the Korean peninsula, Taiwan and the Philippines in the South China Sea, past Singapore and through the Strait of Malacca, to emerge into the Bay of Bengal and then cut across through the Andaman and Nicobar archipelago to Chennai.This sea route covers a distance of approximately 5,600 nautical miles, or about 10,300 km. It will ensure connectivity between the two major ports which will give impetus to the cooperation between India and the Russian Far East. Source:https://vajiramias.com/current-affairs/sea-route-from-chennai-to- vladivostok/5d7348941d5def43bd4c0cf1/ Q66. Which one of the following lakes is the world’s largest man-made lake and reservoir by volume? a. Lake Baikal b. Lake Superior c. Lake Huron d. Lake Kariba Answer: d Explanation: Lake Kariba is the world’s largest man-made lake and reservoir by volume. It lies along the border between Zambia and Zimbabwe. Lake Kariba was filled between 1958 and 1963 following the completion of the Kariba Dam at its northeastern end.Hence, option (d) is the correct answer.

Source:https://vajiramias.com/current-affairs/robert-mugabe/5d7349591d5def43bcef130d/ Q67. Which of the following countries are the eight permanent members of the Shanghai Cooperation Organisation (SCO)? 1. Tajikistan 2. India 3. Pakistan 4. Myanmar Select the correct answer using the code given below: a. 2 and 3 only b. 1 and 2 only c. 1, 2 and 3 only d. 1, 2, 3 and 4 Answer: c Explanation: Shanghai Cooperation Organisation (SCO) is a Eurasian political, economic and military organisation, which was founded in Shanghai by the leaders of China, Russia, Kazakhstan, Kyrgyzstan, and Tajikistan in the year 2001. The aim of SCO is to establish cooperation between member nations on Security-related concerns, Resolving border issues, Military cooperation, Intelligence sharing, Countering terrorism and Countering American influence in Central Asia. Apart from the founding members Uzbekistan joined the group later as a permanent member.India and Pakistan are the newest inclusion to the Organisation and it added another 1.45 billion people in the SCO making the group cover around 40 per cent of the global population.Hence, option (c) is the correct answer.

Source:https://vajiramias.com/current-affairs/conference-of-military-medicine-for-sco- member-states/5d749b6d1d5def43be3fd74f/ Q68. Which of the following are the Miniratna Category – I Public Sector Undertaking of Government of India? 1. BEML Limited 2. Airports Authority of India 3. Bharat Heavy Electricals Limited 4. Steel Authority of India Limited Select the correct answer using the code given below: a. 2 and 3 only b. 1 and 2 only c. 1, 2 and 3 only d. 1, 2, 3 and 4 Answer: b Explanation: Prime Minister of India recently launched Mumbai’s first ‘Make in India’ metro coach, manufactured by the state-run BEML. BEML Limited (formerly Bharat Earth Movers Limited) was established in 1964. It is a ‘Miniratna-Category-1’ Public Sector Undertaking of Government of India. It manufactures a variety of heavy equipment, such as that used for earthmoving, transport and mining. The Company operates under three major Business verticals viz., Mining & Construction, Defence and Rail & Metro. It is headquartered in Bengaluru. BEML has manufacturing plants in Kolar Gold Fields, Bengaluru, Palakkad and Mysore. Bharat Heavy Electricals Limited and Steel Authority of India Limited are Maharatna CPSEs. Airports Authority of India is Miniratna Category - I CPSE.Hence, option (b) is the correct answer. Source:https://vajiramias.com/current-affairs/beml/5d7498cb1d5def43b4c621db/ Q69. With reference to the Mega Food Park project, consider the following statements: 1. It is under the Union Ministry of Agriculture. 2. Under the Scheme, Government of India provides financial assistance upto Rs. 50 Crore per Mega Food Park project. Which of the statements given above is/are correct? a. 1 only b. 2 only c. Both 1 and 2 d. Neither 1 nor 2 Answer: b Explanation: Statement 1 is incorrect: Mega FoodPark’s objective is to give a major boost to the food processing sector by adding value and reducing food wastage at each stage of the supply chain with a particular focus on perishables.Parent ministry of scheme is the Ministry of Food Processing Industries. The Mega Food Park project is implemented by a Special Purpose Vehicle (SPV) which is a Body Corporate registered under the Companies Act.However, State Government, State Government entities and Cooperatives are not required to form a separate SPV for implementation of Mega Food Park project. Statement 2 is correct: Under the Scheme, Government of India provides financial assistance up to Rs. 50 Crore per Mega Food Park project. The Mega Food Park Scheme is based on “Cluster” approach and envisages the creation of state of art support infrastructure in a well-defined Agri / horticultural zone for setting up of modern food processing units along with well-established supply chain. Source:https://vajiramias.com/current-affairs/mega-food- park/5d749a561d5def43be3fd748/ Q70. With reference to the Nilgiri tahr, consider the following statements: 1. It is the state animal of Kerala. 2. Eravikulam National Park is home to its largest population. Which of the statements given above is/are correct? a. 1 only b. 2 only c. Both 1 and 2 d. Neither 1 nor 2 Answer: b Explanation: Statement 1 is incorrect: The population of Nilgiri tahr in the Mukurthi National Park has risen from 568 in 2018 to 612 in 2019. This was the second consecutive year that an increase in the population of the animal had been recorded in the park.They are found at high elevations on cliffs, grass-covered hills and open terrain (i.e. open montane grassland habitats at altitudes of 1,200 – 2,600 meters of the South WesternGhats.Their present distribution is limited to approximately 5% of the Western Ghats in southern India (Kerala and Tamil Nadu). It is the state animal of Tamil Nadu. Statement 2 is correct: Eravikulam National Park is home to its largest population.The population is around 2,500 in Wild.Listed in Schedule I of Wildlife (Protection) Act, 1972 and listed as Endangered on IUCN Red List.Principal threats are habitat loss (mainly from domestic livestock and spread of invasive plants which leads to diminishing grazing land) and poaching.

Mukurthi National Park (MNP)

Mukurthi National Park (MNP) is a protected area located in the northwest corner of TamilNadu. It has montane grasslands and shrublands interspersed with sholas in a high-altitude area of high rainfall. It’s a part of Nilgiri Biosphere Reserve, India’s first International Biosphere Reserve.As part of the Western Ghats, it is also a UNESCO World Heritage Site.

Source:https://vajiramias.com/current-affairs/nilgiri-tahr/5d76072f1d5def43b9ab3fd6/ Q71. Which of the following statement is not correct about the particularly vulnerable tribal groups (PVTGs)? a. PVTGs were earlier known as the Primitive Tribal Groups (PTGs) b. They reside in 18 States and UT of A&N Islands. c. There are 175 PVTGs officially notified in the country so far. d. Saharias are a particularly vulnerable tribal group. Answer: c Explanation: Particularly vulnerable tribal groups (PVTGs) are separately classified by the government of India.There are 75 PVTGs officially notified in the country so far. In 2006, the Government of India renamed the Primitive Tribal Groups (PTGs) as Particularly Vulnerable Tribal Groups (PVTGs). They reside in 18 States and UT of A&N Islands. According to the 12th plan, out of the total ST population, 3 % belong to the PTVGs. These tribal communities have the following features:

• Pre-agricultural system of existence (i.e. the practice of hunting and gathering), or • Zero or negative population growth, or • Extremely low level of literacy in comparison with other tribal groups. Saharias are a particularly vulnerable tribal group. Most of the Saharia (or Sor) of India live in the state of Madhya Pradesh. They may also be located in the hills of the Ganjam district of southern Orissa, as well as in the states of Andhra Pradesh, Bihar, Tamil Nadu, West Bengal, and the Plains division of Assam. They speak a Munda language that belongs to the Austro- Asiatic language family. Hence, option (c) is the correct answer. Source:https://vajiramias.com/current-affairs/saharias/5d7606c91d5def43be400e9f/ Q72. Which of the following pairs is/are correctly matched?

Tribes States

1. Sentinelese Odisha

2. Sangtams Assam

3. Adi Arunachal Pradesh

Select the correct answer using the code given below: a. 3 only b. 2 and 3 only c. 1 and 2 only d. 1, 2 and 3 Answer: a Explanation: Pair (1) is not correctly matched:The Sentinelese is a negrito tribe who live on the North Sentinel Island of the Andamans. Acc. to various estimates, the Sentinelese presence on the islands varies from 2,000 years to 30,000 years ago. Census 2001 counted 39 inhabitants. The Sentinelese are hunter-gatherers, likely using bows and arrows to hunt and more rudimentary methods to catch local seafood. It is known that the Sentinelese speak their own language, the Sentinelese language. The Sentinelese have been fiercely hostile to outside contact. They have been mostly left alone even from colonial times, unlike other tribes such as the Onges, Jarawas and Great Andamanese, because the land they occupy has little commercial attraction. They are designated as a Scheduled Tribe. Access to North Sentinel Island and its buffer zone is strictly restricted under the Protection of Aboriginal Tribe (Regulation), 1956 and Regulations under Indian Forest Act, 1927. Under the Andaman and Nicobar Islands (Protection of Aboriginal Tribes) Regulation, 1956, traditional areas occupied by the tribes have been declared as reserves, Entry of persons and Photographing the tribe members is prohibited except with authorisation. Any passage within three miles of the coastline of Island is illegal, and is enforced by the Indian Navy. Pair (2) is not correctly matched:The Sangtamsare one of the major Naga tribe living in the Tuensang and Kiphire districts of Nagaland. Mongmong Festival is one of the popular festivals of Nagaland celebrated by Sangtam tribe. Pair (3) is correctly matched:The Adi people are one of the most populous groups of indigenous peoples in the Indian state of Arunachal Pradesh. Source:https://vajiramias.com/current-affairs/sentinelese- tribe/5d7607821d5def43b38f0ac0/ Q73. Consider the following statements: 1. It is a landlocked multiethnic country located in the heart of south-central Asia. 2. The Hindu Kush mountains divide the northern provinces from the rest of the country. 3. It is one of the countries invaded by the Soviet Union during the Cold War decades. Above statements describes which one of the following countries? a. Iraq b. Iran c. Turkmenistan d. Afghanistan Answer: d Explanation: Afghanistan is a landlocked multiethnic country located in the heart of south-central Asia.The Hindu Kush mountains that run northeast to southwest divide the northern provinces from the rest of the country.The country served as a buffer between the British and Russian Empires until it won independence from notional British control in 1919. The Soviet Union invaded in 1979 to support the tottering Afghan communist regime, touching off a long and destructive war. The USSR withdrew in 1989 under relentless pressure by internationally supported anti-communist mujahidin rebels.Hence, option (d) is the correct answer.

Source:https://vajiramias.com/current-affairs/us-draft-troop-pullout-deal-with- taliban/5d7607bc1d5def43b9ab3fe0/ Q74. What is 'Golden Arrows', recently seen in news? a. Resurrected -based 17 Squadron Air Force Station. b. Israel's new anti-missile system c. A mobile all-weather air defence system developed by Russia d. None of the above Answer: a Explanation: The Indian Air Force (IAF) resurrected the Air Force Station Ambala-based 17 Squadron 'Golden Arrows', which will operate the first squadron of Rafale fighter jets in the near future. The squadron was formed at Ambala in 1951 under the command of Flight Lieutenant D L Springett and was then equipped with Harvard-II B aircraft. The Squadron converted to the Mig- 21 M in 1975. In 1988, the Squadron was presented colours by then President of India R Venkataraman. Under the command of then Wing Commander BS Dhanoa, Golden Arrows participated actively in Operation SafedSagar during the Kargil conflict in 1999.The squadron was disbanded in 2016, but now has been resurrected.In the near future, 17 Squadron will be the first Squadron to be equipped with the State of the art Rafale aircraft, which is an extremely capable, fourth generation, multirole aircraft with advanced weapons.Hence, option (a) is the correct answer. Source:https://vajiramias.com/current-affairs/golden-arrows/5d79adc81d5def661d826874/ Q75. Which of the following are the members of the Association of Southeast Asian Nations (ASEAN)? 1. Brunei 2. Cambodia 3. Indonesia 4. Malaysia Select the correct answer using the code given below: a. 1 and 4 only b. 1 and 2 only c. 1, 2 and 3 only d. 1, 2, 3 and 4 Answer: d Explanation: The 10 ASEAN members are Brunei, Cambodia, Indonesia, Malaysia, Myanmar, Singapore, Thailand, the Philippines, Laos and Vietnam.Hence, option (d) is the correct answer.Economic Ministers from ten ASEAN Member States and the Minister of Commerce of India recently met at Bangkok (Thailand) for the 16th AEM-India Consultations. Key highlights of 16th AEM-India Consultations: 1. During the meeting, India and ASEAN agreed to review their Free Trade Agreement (FTA) in goods to make it more business-friendly and boost economic ties. The two sides also decided to constitute a joint committee for this purpose. 2. The Ministers noted that the two-way merchandise trade between ASEAN and India grew by 9.8 per cent from USD 73.6 billion in 2017 to USD 80.8 billion in 2018. There is also recovery of Foreign Direct Investment (FDI) inflows from India in 2018, amounting to USD 1.7 billion. 3. This placed India as ASEAN’s sixth largest trading partner and sixth largest source of FDI among ASEAN Dialogue Partners. The India-ASEAN FTA in goods was signed in 2009 and came into force from January 2010. Under the pact, two trading partners set timelines for eliminating duties on the maximum number of goods traded between the two regions.

Source:https://vajiramias.com/current-affairs/asean-us-maritime-exercise- aumx/5d6e15c71d5def5e3719e4f0/ Q76. With reference to the Market Intervention Scheme (MIS), consider the following statements: 1. All the agricultural and horticultural commodities for which Minimum Support Price (MSP) is not fixed and are generally perishable in nature are covered under MIS. 2. Under MIS, funds are allocated to the States. Which of the statements given above is/are correct? a. 1 only b. 2 only c. Both 1 and 2 d. Neither 1 nor 2 Answer: a Explanation: Market Intervention Scheme (MIS) is implemented by the Department of Agriculture & Cooperation (Government of India). All the agricultural and horticultural commodities for which Minimum Support Price (MSP) are not fixed and are generally perishable in nature are covered under MIS. The basic objective of MIS is to provide remunerative prices to the farmers in case of a glut in production and fall in prices. Further, the MIS is implemented on the specific requests of the State Government/UT Administrations willing to share the loss with Central Government on 50:50 basis (75:25 in case of North-Eastern States). Under MIS, funds are not allocated to the States. Instead, central share of losses as per the guidelines of MIS is released to the State Governments/UTs, for which MIS has been approved based on specific proposals received from them. Hence only statement 1 is correct. Source:https://vajiramias.com/current-affairs/market-intervention-scheme- mis/5d79b0471d5def661c076815/ Q77. Which of the following is/are the initiative(s) launched by MHRD’s Innovation Cell (MIC) and AICTE to boost research and innovation in the country? 1. Atal Ranking of Institutions on Innovation Achievements (ARIIA) 2020. 2. Smart India Hackathon 2020 (SIH2020). 3. Protsahan Mudra Scheme. Select the correct answer using the code given below: a. 1 only b. 1 and 2 only c. 2 and 3 only d. 1, 2 and 3 Answer: d Explanation: Union Human Resource Development (HRD) Minister recently launched several initiatives of MIC and AICTE to boost research and innovation in the country. Union HRD Minister attended the First Annual Innovation Festival of the MHRD’s Innovation Cell (MIC). MIC organized the first Annual Innovation Festival in coordination with AICTE. In 2018, the MHRD, initiated the separate Innovation cell with support from AICTE to foster the culture of innovation in all educational institutions of India. Initiatives Launched:

• Launch of Institution’s Innovation Council (IIC) 2.0 from July 2019 – June 2020: MHRD’s Innovation Cell has envisioned establishing a strong network of Institution’s Innovation Council (IICs) in Higher Educational Institutes to promote innovation and start-up in academia. • Launch of Atal Ranking of Institutions on Innovation Achievements (ARIIA) 2020: ARIIA is an initiative of Ministry of Human Resource Development (MHRD) to systematically rank all major educational institutions and universities on indicators related to the promotion of “Innovation and Entrepreneurship Development” amongst their students and faculties. • Release of guidelines on ‘National Innovation and Start-up Policy 2019’ for students and faculty of all higher education institutions in India who are keen to become entrepreneurs. • Launch of Smart India Hackathon 2020 (SIH2020): Smart India Hackathon is a flagship program and nationwide initiative by MHRD’s Innovation Cell (MIC) and AICTE to provide students a platform to solve some of the pressing problems we face in our daily lives. • Announcement of new ATAL ACADEMIES: It is proposed to set up 10 (ten) more AICTE Training And Learning (ATAL) Academies/Centres (apart from existing 4). • Launch of Protsahan Mudra Scheme: Protsahan Mudra is real-time rewards Engine to give rewards to Students and teachers daily or as close to the action that qualifies for the reward. • Announcement of AICTE – VISHWAKARMA AWARDS’ 2019: AICTE has introduced Vishwakarma Awards from 2017 to encourage and motivate young students and institutions to raise their performance in their specific domains. The theme of Vishwakarma Award 2019 is “How to enhance the income of Village”. Hence, option (d) is the correct answer. Source:https://vajiramias.com/current-affairs/initiatives-of-mic-and-aicte-to-boost- research-and-innovation-in-the-country/5d7afe7e1d5def055b0c59fd/ Q78. With reference to the Global Antimicrobial Resistance (AMR) Research and Development (R&D) Hub, consider the following statements: 1. India has joined the Global Antimicrobial Resistance (AMR) Research and Development (R&D) Hub as a new member. 2. Its operations are currently financed through grants from the German Federal Ministry of Education and Research (BMBF) and the Federal Ministry of Health (BMG). Which of the statements given above is/are correct? a. 1 only b. 2 only c. Both 1 and 2 d. Neither 1 nor 2 Answer: c Explanation: Statement 1 is correct: India has joined the Global Antimicrobial Resistance (AMR) Research and Development (R&D) Hub as a new member. The Global AMR R&D Hub was launched in May 2018 in the margins of the 71st session of the World Health Assembly, following a call from G20 Leaders in 2017. The Global AMR R&D Hub supports global priority setting and decision-making on the allocation of resources for AMR R&D, the potential for cross-sectoral collaboration and leveraging in AMR R&D. Secretariat is in Berlin, Germany. Statement 2 is correct: It’s operations are currently financed through grants from the German Federal Ministry of Education and Research (BMBF) and the Federal Ministry of Health (BMG). Inclusion of India has expanded collaboration in global AMR R&D to 16 countries, the European Commission, two philanthropic foundations and four international organisations (as observers). Source:https://vajiramias.com/current-affairs/global-antimicrobial-resistance-amr- research-and-development-rd-hub/5d7c548b1d5def0d2a4cf073/ Q79. Which of the following is the only exoplanet known to have both water and temperatures that could support life? a. K2-18b b. Kepler- 20e c. Kepler-22b d. LHS1140b Answer: a Explanation: Astronomers have for the first time discovered water vapour in the atmosphere of a distant planet called K2-18b. Astronomers at the University College London used data from NASA's Hubble Space Telescope to make the discovery. They published their findings in the journal Nature Astronomy. It is the first time that they have detected water on a planet in the habitable zone around a star where the temperature is potentially compatible with the presence of life. K2- 18b has an atmosphere and it has water in it, thus making it the only exoplanet known to have both water and temperatures that could support life. This exoplanet, known as K2-18b, orbits a red dwarf star 110 light-years away from Earth in the Leo constellation of the Milky Way. An exoplanet refers to a planet beyond our solar system. K2-18b is 8 times the mass of earth and twice as big. It is known as a super-Earth, or exoplanets between the mass of Earth and Neptune. The planet was first discovered in 2015 by NASA's Kepler spacecraft. It resides in a habitable zone — the region around a star in which liquid water could potentially pool on the surface of a rocky planet. Hence, option (a) is the correct answer. Source:https://vajiramias.com/current-affairs/k2-18b/5d7c56371d5def0d2a4cf0b2/ Q80. With reference to the Ekalavya Model Residential Schools, consider the following statements: 1. The scheme of EMRSs was introduced in the year 1997-98 to ensure tribal students get access to quality education in the remote tribal areas. 2. It is implemented by the Union Ministry of Human Resource Development. Which of the statements given above is/are correct? a. 1 only b. 2 only c. Both 1 and 2 d. Neither 1 nor 2 Answer: a Explanation: The Prime Minister recently launched 462 Ekalavya Model Schools across the country in Tribal Dominated Areas. It is implemented by the Ministry of Tribal Affairs. The scheme of EMRSs was introduced in the year 1997-98 to ensure tribal students get access to quality education in the remote tribal areas. The schools would focus to provide quality upper primary, secondary and senior secondary level education to ST Students in these areas. Apart from school building including hostels and staff quarters, provision for a playground, students' computer lab, teacher resource room etc. are also included in the EMRSs. Hence only statement 1 is correct. Value Addition: In December 2018, Union Cabinet revamped the scheme with following features – EMRSs will be set up in every block with more than 50% ST population and at least 20,000 tribal persons. Thereby around 462 schools will be set up in addition to the 288 EMRSs already sanctioned. There will be an Autonomous Society under the Ministry of Tribal Affairs similar to NavodayaVidyalayaSamiti to run the EMRSs. Source:https://vajiramias.com/current-affairs/eklavya-model-residential- schools/5d7c58ac1d5def0d2ea0b56e/ Q81. Which one of the following statements is not correct about the National Pension Scheme for Traders and Self-Employed Persons? a. It is a voluntary pension scheme for the Vyaparis with annual turnover not exceeding Rs 1.5 crore. b. It is a scheme for entry age of 18 to 50 years c. The Central Government makes 50 % of the contribution and the remaining 50% is made by the beneficiary. d. At the time of enrollment, the beneficiary is required to have an Aadhaar card Answer: b Explanation: National Pension Scheme for Traders and Self Employed Persons is a voluntary and contributory pension scheme for the Vyaparis (shopkeepers/retail traders and self-employed persons) with annual turnover not exceeding Rs 1.5 crore. It is a scheme for entry age of 18 to 40 years with a provision for minimum assured pension of Rs 3,000/- monthly on attaining the age of 60 years. The Central Government shall give a 50 % share of the monthly contribution and the remaining 50% contribution shall be made by the beneficiary. With this nation-wide launch, people can enrol through 3.50 lakh Common Service Centres (CSCs) across the country. In addition people can also self-enrol by visiting the portal www.maandhan.in/vyapari. Criteria for enrolment:

• At the time of enrollment, the beneficiary is required to have an Aadhaar card and a saving bank/ Jan-Dhan Account passbook only. • He/ She should be within 18 to 40 years of age group. • GSTIN is required only for those with turnover above Rs. 40 lakhs. • The beneficiary should not be income taxpayer and also not a member of EPFO/ESIC/NPS (Govt.)/PM-SYM. Target: this scheme will target enrolling 25 lakh subscribers in 2019-20 and 2 crore subscribers by 2023-2024. An estimated 3 croreVyaparis in the country are expected to be benefitted under the pension scheme. Hence, option (b) is the correct answer. Source:https://vajiramias.com/current-affairs/national-pension-scheme-for-traders-and- self-employed-persons/5d7c59f51d5def0d2951c5ed/ Q82. With reference to the Pradhan Mantri Kisan Maan Dhan Yojana, consider the following statements: 1. It is an old-age pension scheme for all landholding Small and Marginal Farmers (SMFs) in the country. 2. The Life Insurance Corporation of India (LIC) is the Pension Fund Manager and responsible for Pension payout. Which of the statements given above is/are correct? a. 1 only b. 2 only c. Both 1 and 2 d. Neither 1 nor 2 Answer: c Explanation: Pradhan Mantri Kisan Maan-Dhan Yojana (PM-KMY) is an old-age pension scheme for all landholding Small and Marginal Farmers (SMFs) in the country. It is voluntary and contributory for farmers in the entry age group of 18 to 40 years and a monthly pension of Rs. 3000/- will be provided to them on attaining the age of 60 years. The Life Insurance Corporation of India (LIC) shall be the Pension Fund Manager and responsible for Pension payout. If the farmer dies after the retirement date, the spouse will receive 50% of the pension as Family Pension. After the death of both the farmer and the spouse, the accumulated corpus shall be credited back to the Pension Fund. Eligibility: 1. Small and Marginal Farmer (SMF) - a farmer who owns cultivable land up to 2 hectares as per land records of the concerned State/UT. 2. Age of 18- 40 years. The following categories of farmers have been brought under the exclusion criteria: 1. SMFs covered under any other statuary social security schemes such as National Pension Scheme (NPS), Employees’ State Insurance Corporation scheme, Employees’ Fund Organization Scheme etc. 2. Farmers who have opted for Pradhan MantriShram Yogi MaanDhanYojana (PM-SYM) and Pradhan MantriLaghuVyapariMaan-dhanYojana (PM-LVM). 3. All Institutional Landholders; and 4. Former and present holders of constitutional posts. Hence both statements are correct. Source:https://vajiramias.com/current-affairs/pradhan-mantri-kisan-maan-dhan-yojana- pm-kmy/5d7c5a7d1d5def0d289d9629/ Q83. Which of the following is the aim of ‘4P1000 Initiative’, often mentioned in news? a. To demonstrate that an annual growth rate of 0.4% in the soil carbon stocks, in the first 30-40 cm of agricultural soil, would significantly reduce the CO2 concentration in the atmosphere related to human activities. b. First holistic program for the empowerment of 1000 women in Africa by integrating them in 4Ps - politics, people’s movement, planet conservation and public service. c. To reduce petroleum consumption by 4% with the help of promotion of 1000 electric cars in each Nagarpalikas. d. None of the above. Answer: a Explanation: The international initiative “4per1000”, launched by France in 2015 at the COP 21, consists of federating all voluntary stakeholders of the public and private sectors under the framework of the Lima-Paris Action Plan (LPAP). The aim of the initiative is to demonstrate that an annual growth rate of 0.4% in the soil carbon stocks, in the first 30-40 cm of agricultural soil, would significantly reduce the CO2 concentration in the atmosphere related to human activities. The Executive Secretariat of the "4 per 1000" initiative is hosted by the CGIAR System Organization, an international organization based in Montpellier. Hence, option (a) is the correct answer. Sourcehttps://vajiramias.com/current-affairs/4p1000-initiative- ticd/5d7dc3001d5def0d29521dd2/ Q84. Survey of India is the National Survey and Mapping Organization of the country. It is under: a. Ministry of Statistics and Programme Implementation. b. Ministry of Science and Technology c. Ministry of Mines d. Ministry of Earth Sciences Answer: b Explanation: Survey of India is the National Survey and Mapping Organization of the country. It has the responsibility to ensure that the country's domain is explored and mapped suitably. It also provides base maps for expeditious and integrated development.It is under the Department of Science & Technology, Ministry of Science and Technology. It is the oldest scientific department of the Govt. of India. It was set up in 1767. Headquarters is at Dehradun, Uttarakhand. Hence, option (b) is the correct answer.

The Great Trigonometrical Survey

It was a project which aimed to measure the entire Indian subcontinent with scientific precision. It was begun in 1802 by the infantry officer William Lambton, under the auspices of the East India Company. Under the leadership of his successor, George Everest, the project was made a responsibility of the Survey of India.

Among the many accomplishments of the Survey were the demarcation of the British territories in India and the measurement of the height of the Himalayan giants: Everest, K2, and Kanchenjunga. The Survey had an enormous scientific impact as well, being responsible for one of the first accurate measurements of a section of an arc of longitude, and for measurements of the geodesic anomaly which led to the development of the theories of isostasy.

Source:https://vajiramias.com/current-affairs/survey-of-india/5d7f157a1d5def2401a3121f/ Q85. Which of the following vulture species are considered as Critically Endangered? 1. Slender-billed Vulture 2. White-rumped vulture 3. Indian Vulture 4. Griffon Vulture Select the correct answer using the code given below: a. 1 and 4 only b. 1 and 2 only c. 1, 2 and 3only d. 1, 2, 3 and 4 Answer: c Explanation: Three of India's resident vulture species - the Slender-billed Vulture Gyps tenuirostris, White-rumped vulture Gyps bengalensis and Indian Vulture Gyps indicus - are considered Critically Endangered. Griffon Vultureis listed as least concern. These vulture species have witnessed a catastrophic dip in population numbers by more than 90 percent since the 1990s, putting them on the International Union for Conservation of Nature (IUCN’s) list of critically endangered species. The major reason behind the vulture population getting nearly wiped out was the drug Diclofenac, found in the carcass of cattle the vultures fed on. The drug, whose veterinary use was banned in 2008, was commonly administered to cattle to treat inflammation. To study the cause of deaths of vultures, a Vulture Care Centre (VCC) was set up at Pinjore, Haryana in 2004. At present there are nine Vulture Conservation and Breeding Centres (VCBC) in India, of which three are directly administered by Bombay Natural History Society (BNHS). The objective of the VCBCs is not only to look after the vultures and breed them in captivity, but also to release them into the wild. The total number of vultures in these VCBCs is now more than 700. Hence, option (c) is the correct answer. Source:https://vajiramias.com/current-affairs/vulture-conservation-and-breeding-centres- vcbc/5d7f17231d5def23ff4dc9d7/ Q86. Which of the following pairs is/are correctly matched?

Protected areas States

1. Amrabad Tiger Reserve Kerala

2. Bandhavgarh National Park Maharashtra

3. Periyar National Park Tamil Nadu

Select the correct answer using the code given below: a. 1 only b. 1 and 2 only c. 2 and 3 only d. None of the above Answer: d Explanation: Point (1) is not correct:Amrabad Tiger Reserve lies in Nallamala hills of Telangana state.It is India’s second-largest tiger reserve, next only to NagarjunasagarSrisailam Tiger Reserve. It has a large presence of the Chenchu tribe.

Point (2) is not correct:Periyar National Park and Wildlife Sanctuary (PNP) is a protected area near Thekkady in the state of Kerala.

Point (3) is not correct:Bandhavgarh National Park is in the central Indian state of Madhya Pradesh. This biodiverse park is known for its large population of royal Bengal tigers, especially in the central Tala zone. Hence, option (d) is the correct answer.

Source:https://vajiramias.com/current-affairs/amrabad-tiger-reserve- atr/5d8049761d5def270db45757/ Q87. Which of the following comes under the ten threats to global health in 2019 according to the World Health Organization (WHO)? 1. Vaccine hesitancy 2. Air pollution and climate change 3. Weak primary health care Select the correct answer using the code given below: a. 1 only b. 1 and 2 only c. 2 and 3 only d. 1, 2 and 3 Answer: d Explanation: The World Health Organization (WHO) has included ‘vaccine hesitancy’ as one of the 10 threats to global health for the year 2019.Vaccine hesitancy means the reluctance or refusal to vaccinate despite the availability of vaccines. Vaccine hesitancy threatens to reverse progress made in tackling vaccine-preventable diseases. Measles, for example, has seen a 30% increase in cases globally in 2018. Ten threats to global health in 2019 according to WHO are:

1. Air pollution and climate change 2. Noncommunicable diseases 3. Global influenza pandemic 4. Fragile and vulnerable settings 5. Antimicrobial resistance 6. Ebola and other high-threat pathogens 7. Weak primary health care 8. Vaccine hesitancy 9. Dengue 10. HIV

Hence, option (d) is the correct answer.

Source:https://vajiramias.com/current-affairs/vaccine- hesitancy/5d8306a01d5def2711d433db/

Q88. NIRVIK Scheme is an initiative of which of the following Union Ministries? a. Ministry of Finance b. Ministry of Commerce & Industry c. Ministry of Social Justice and Empowerment d. Ministry of Law and Justice Answer: b Explanation: Ministry of Commerce & Industry has recently introduced NIRVIK Scheme to enhance loan availability and ease the lending process. Hence, option (b) is the correct answer. Salient features of the scheme:

• Ministry of Commerce & Industry through Export Credit Guarantee Corporation (ECGC) has introduced a new Export Credit Insurance Scheme (ECIS) called NIRVIK to enhance loan availability and ease the lending process. • Under ECIS, the insurance cover percentage has also been enhanced to 90% from the present average of 60% for both Principal and Interest of the loan for pre- and post- shipment credit. • Claim inspection would be waived for up to Rs 10 crore. For claims higher than this amount, an inspection of bank documents and records by ECGC officials will be mandatory. • The existing premium rate would be lowered, with loans to be categorised into two broad categories of those below Rs 80 crore and those above that amount. Loans above Rs 80 crore will be further divided into those that are not for gold, jewellery or diamonds, and those that are. • Public sector banks had agreed to give the ECGC-covered loans at a rate that is a maximum of 2.2% higher than the prevailing repo rate, and this would be adjusted as the repo rate changes.

Source:https://vajiramias.com/current-affairs/nirvik/5d83051c1d5def2711d43362/

Q89. Which among the following are Scorpene class stealth submarines being built under the Project 75? 1. INS Karanj 2. INS Vikrant 3. INS Khanderi 4. INS Vela Select the correct answer using the code given below: a. 1 and 2 only b. 1 and 3 only c. 2, 3 and 4 only d. 1, 3 and 4 only Answer: d Explanation: Six Scorpene class submarines are being built under Project 75. The submarines are designed by French naval defence and energy group DCNS and manufactured by the Mazagon Dock Shipbuilders Limited (MDSL), Mumbai under a 2005 contract worth $3.75 billion.

• INS Kalvari: It was commissioned in December 2017 by PM Narendra Modi. • INS Khanderi: It was launched in January 2017, and commissioned in September 2019. • INS Karanj: It was launched in January 2018. The fourth Scorpene, VELA was recently launched in May 2019, and is being prepared for sea trials. The remaining two submarines, VAGIR and VAGSHEER, are in various stages of outfitting. The last of the submarines will be delivered by 2022. • INS Vikrant is the first aircraft carrier built in India and the first aircraft carrier built by Cochin Shipyard in Kochi for the Indian Navy. Hence option (d) is correct. Source: https://vajiramias.com/current-affairs/khanderi/5d84625d1d5def270db55231/ Q90. With reference to the Pradhan Mantri Matru Vandana Yojana (PMMVY), consider the following statements: 1. Ministry of Health and Family Welfare is implementing Centrally Sponsored Pradhan Mantri Matru Vandana Yojana (PMMVY). 2. Under it, Pregnant Women and Lactating Mothers receive a conditional cash benefit of a certain amount in instalments. Which of the statements given above is/are correct? a. 1 only b. 2 only c. Both 1 and 2 d. Neither 1 nor 2 Answer: b Explanation: Statement 1 is incorrect: The Ministry of Women and Child Development is implementing Centrally Sponsored Pradhan MantriMatruVandanaYojana (PMMVY). Implementation of the scheme started with effect from 01.01.2017. PMMVY is a direct benefit transfer (DBT) scheme under which cash benefits are provided to pregnant women in their bank account directly to meet enhanced nutritional needs and partially compensate for wage loss. Statement 2 is correct: Under the ‘Scheme’, Pregnant Women and Lactating Mothers (PW&LM) receive a cash benefit of Rs. 5,000 in three instalments on fulfilling the respective conditionality, viz. early registration of pregnancy, ante-natal check-up and registration of the birth of the child and completion of the first cycle of vaccination for the first living child of the family. The eligible beneficiaries also receive cash incentive under JananiSurakshaYojana (JSY). Thus, on average, a woman gets Rs. 6,000.

Source:https://vajiramias.com/current-affairs/pradhan-mantri-matru-vandana-yojana- pmmvy/5d85b2941d5def270be30ce2/ Q91. Facilitation through Margadarshan and Margadarshak is an initiative of: a. All India Council for Technical Education(AICTE) b. University Grants Commission (UGC) c. Medical Council of India (MCI) d. Central Board of Secondary Education Answer: a

Explanation:

Union Human Resource Development Minister launched several initiatives of All India Council for Technical Education (AICTE) including Facilitation through Margadarshan and Margadarshak. Under the initiative of Facilitation through Margadarshan and Margadarshak, the topmost institutions will mentor other institutions, so that they can improve their rankings and follow best practices of the mentor institute. Margadarshan:

• Under this scheme, institutions having good accreditation record / highly performing institutions are supposed to mentor relatively newer 10 - 12 potential institutions. • These institutions are also provided funding uptoRs. 50 lakhs per institution over a period of three years in instalments for carrying out various activities like trainings, workshops, conferences and travel. Margdarshak:

• Under this scheme, mentor teachers or Margdarshaks who are either serving or superannuated but willing and motivated with good knowledge of accreditation and who can devote adequate time to make required visits to these Institutions are identified. • These Margdarshaks will regularly visit to the mentee institutions, stay on their campus and guide them for their improvement in quality so that institutions are able to get accreditation by NBA. Hence, option (a) is the correct answer.

Source:https://vajiramias.com/current-affairs/facilitation-through-margadarshan-and- margadarshak/5d85b3e01d5def270be30d1b/

Q92. With reference to PACEsetter fund programme, consider the following statements: 1. It was constituted by France in 2015 at the COP 21, United Nations Climate Change Conference. 2. Its objective is to provide early-stage grant funding to accelerate the commercialization of innovative off-grid clean energy products, systems, and business models. Which of the statements given above is/are correct? a. 1 only b. 2 only c. Both 1 and 2 d. Neither 1 nor 2 Answer: b Explanation: PACEsetter fund programme was constituted by India and the USA in 2015. It is a joint fund to provide early-stage grant funding to accelerate the commercialization of innovative off-grid clean energy products, systems, and business models. The Fund's main purpose is to improve the viability of off-grid renewable energy businesses that sell small scale (under 1 megawatt) clean energy systems to individuals and communities without access to grid-connected power or with limited/intermittent access (less than 8 hours per day). Hence only statement 2 is correct. Source:https://vajiramias.com/current-affairs/pacesetter-fund- programme/5d8709f61d5def270e2128cb/ Q93. Kodiakkarai Wild Life sanctuary, recently in news, is located in which of the following states? a. Tamil Nadu b. Kerala c. Andhra Pradesh d. Karnataka Answer: a Explanation: Kodiakkarai Wild Life sanctuary is also known as Point Calimere Wildlife and Bird Sanctuary (PCWBS). It is a protected area in Tamil Nadu, along the Palk Strait where it meets the Bay of Bengal at Point Calimere (Kodiakkarai) at the tip of Nagapattinam District. The sanctuary was created in 1967 for the conservation of the near-threatened blackbuck antelope, an endemic mammal species of India. Kodiakkarai Wild Life sanctuary was in news due to cohabitating deer population. Hence, option (a) is the correct answer. Source:https://vajiramias.com/current-affairs/kodiakkarai-wild-life- sanctuary/5d8705ed1d5def270e212822/ Q94. Recently Prime Minister of India inaugurated the Gandhi Solar Park at: a. United Nations (UN) headquarters b. Rashtrapati Bhavan c. Longyangxia Dam, China d. Pavagada, Karnataka Answer: a Explanation: Prime Minister of India recently inaugurated the Gandhi Solar Park at the United Nations (UN)headquarters on the occasion of Gandhi's 150th birth anniversary. The 50 kWh roof-top solar park has 193 solar panels—each representing a member of the multilateral body.Hence, option (a) is the correct answer. Source:https://vajiramias.com/current-affairs/gandhi-solar- park/5d8afad21d5def270f66ae75/ Q95. With reference to ‘Sardar Patel National Unity Award’, consider the following statements: 1. It is the highest civilian award in India in the field of contribution to the unity and integrity of India. 2. Not more than three Awards would be given in a year. Which of the statements given above is/are correct? a. 1 only b. 2 only c. Both 1 and 2 d. Neither 1 nor 2 Answer: c Explanation: Statement 1 is correct: Government of India has recently instituted ‘Sardar Patel National Unity Award’ - Highest Civilian Award for contribution to Unity and Integrity of India. The Award, in the name of SardarVallabhbhai Patel, is the highest civilian award in India in the field of contribution to the unity and integrity of India. It seeks to recognize contributions to promote the cause of national unity and integrity. Statement 2 is correct: An Award Committee would be constituted by the Prime Minister, which would include the Cabinet Secretary, Principal Secretary to the PM, Secretary to the President, Home Secretary as Members and three-four eminent persons selected by the PM. All citizens, without distinction of religion, race caste, gender, place of birth, age or occupation, and any institution/organization would be eligible for the Award. Not more than three Awards would be given in a year. It would not be conferred posthumously except in very rare and highly deserving cases. Source:https://vajiramias.com/current-affairs/sardar-patel-national-unity- award/5d8c46371d5def270e224b8a/ Q96. What is ‘2I/Borisov’, recently seen in news? a. The second interstellar object has been officially detected in our solar system. b. First-ever simulated space warfare exercise. c. The new all-sky-survey satellite launched by Israel. d. A first-line HIV drug recommended by the World Health Organisation (WHO). Answer: a Explanation: According to the International Astronomical Union (IAU), the second interstellar object has been officially detected in our solar system.In 2I/Borisov, I stands for "interstellar", and 2 for being the second interstellar object ever discovered, after 'Oumuamua, which was detected in October 2017. The name Borisov honors its discoverer, Crimean amateur astronomer Gennady Borisov.2I/Borisov is the first observed interstellar comet and second observed interstellar Object, after ʻOumuamua.Its chemical composition is similar to the Solar System's long-period comets that originate in the distant OortCloud, rather than the short-period comets that come from closer in.Hence, option (a) is the correct answer. Source:https://vajiramias.com/current-affairs/2iborisov/5d8d9dd41d5def37c1bce784/ Q97. Which one of the following is the aim of Union government’s ‘mCessationProgramme’, recently seen in news? a. Initiative to quit tobacco by utilising mobile technology b. Address the major causes of mortality among women and children c. Prevention and control of Vector-Borne Diseases (VBDs) d. Early detection of Leprosy Answer: a Explanation: The Central government’s ‘mCessation’ Programme to quit tobacco is a text messaging programme for mobile phone users. It is part of the union government’s Digital India initiative. A person looking to quit tobacco can give a missed call to 011-22901701 after which, they will be sent a series of messages over several months. In a 2018 report published by the peer- reviewed online journal BMJ Innovations it was reported that the ‘mCessation’ programme in India had seen a 19% quit rate. Hence, option (a) is the correct answer. Source:https://vajiramias.com/current-affairs/tobacco- quitline/5d8ef3fb1d5def37be6f3c0a/ Q98. Which one of the following statements best describes Section 35A of the Banking Regulation Act, 1949, recently seen in news? a. It vests power in the RBI to give directions to banks and take action. b. It vests power in the central government to appoint the central bank governor. c. It empowers the central government to issue directions to the RBI in the public interest. d. It enables RBI to prescribe CRR for scheduled banks without any floor or ceiling rate. Answer: a Explanation: The Banking Regulation Act legislated in the year 1949 comprises a set of rules which govern the banking sector in India. The act vests powers in the RBI to grant licenses to banks as well as work as a banking regulator in India. Section 35A of the Banking Regulation Act, 1949 vests power in the RBI to give directions to banks and can take action, "to prevent the affairs of any banking company being conducted in a manner detrimental to the interests of the depositors or in a manner prejudicial to the interests of the banking company". The RBI under the act can also impose restrictions on banks to ensure better governance and control. Meanwhile, Section 56 of the act is applicable to cooperative societies. Hence, option (a) is the correct answer. Source:https://vajiramias.com/current-affairs/section-35a-of-the-banking-regulation-act- 1949-aacs/5d8ef5471d5def37c3838475/ Q99. ‘Reporters Without Borders’, often in the news, is: a. a non-governmental organization with consultant status at the United Nations. b. an inter-governmental international organization sponsored by G20. c. an inter-governmental agency sponsored by the European Union. d. a specialized agency of the United Nations. Answer: a Explanation: Reporters Without Borders is also known under its original name Reporters Sans Frontières (RSF). It is a non-profit organization, non-governmental organization with consultant status at the United Nations. Headquarters is in Paris, France. Its main aim is to conduct political advocacy on issues relating to freedom of information and freedom of the press. Its key Publications are World Press Freedom Index, Predators of Press Freedom ad Press Freedom Barometer. Hence, option (a) is the correct answer. Source:https://vajiramias.com/current-affairs/fake-news/5d8ef9011d5def37c3838530/ Q100. What is “INS ‘Nilgiri’, recently seen in news? a. Integrated training establishment in the Indian Navy. b. Diesel- Electric submarine of the Indian Navy. c. Nuclear powered ballistic missile submarines. d. New stealth frigates under Project 17A. Answer: d Explanation: INS ‘Nilgiri’ is the first of the Navy's seven new stealth frigates under Project 17A. Project 17A frigates is a design derivative of the Shivalik class stealth frigates with much more advanced stealth features and indigenous weapons and sensors. These frigates are being built using integrated construction methodology. The P17A frigates incorporate new design concepts for improved survivability, sea keeping, stealth and ship manoeuvrability.Hence, option (d) is the correct answer. Source:https://vajiramias.com/current-affairs/ins-nilgiri/5d9043361d5def52bcf5b33a/